Download as pdf or txt
Download as pdf or txt
You are on page 1of 55

https://www.pdfnotes.

co/

O
C
S.
TE
O
FN
D
.P
W
W
W
https://www.pdfnotes.co/

DO NOT OPEN THIS TEST BOOKLET UNTIL YOU ARE ASKED TO DO SO

PPP-PTS-4392/082023/17

GENERAL STUDIES

CURRENT AFFAIRS-2
JULY - AUGUST 2023
Time Allowed: Two Hours Maximum Marks: 200

INSTRUCTIONS

1. IMMEDIATELY AFTER THE COMMENCEMENT OF THE EXAMINATION, YOU SHOULD CHECK THAT
THIS TEST BOOKLET DOES NOT HAVE ANY UNPRINTED OR TORN OR MISSING PAGES OR ITEMS,
ETC. IF SO, GET IT REPLACED BY A COMPLETE TEST BOOKLET.
2. Please note that it is the candidate’s responsibility to encode and fill in the Roll Number carefully
without any omission or discrepancy at the appropriate places in the OMR Answer Sheet. Any
omission/discrepancy will render the Answer Sheet liable for rejection.
3. You have to enter your Roll Number on the test booklet in the Box provided
alongside. DO NOT write anything else on the Test Booklet.
4. This Test Booklet contains 100 items (questions). Each item is printed in English. Each item comprises four
responses (answers). You will select the response which you want to mark on the Answer Sheet. In case you
feel that there is more than one correct response, mark the response which you consider the best. In any case,
choose ONLY ONE response for each item.
5. You have to mark all your responses ONLY on the separate Answer Sheet provided. See directions in the
Answer Sheet.
6. All items carry equal marks.
7. Before you proceed to mark in the Answer Sheet in response to various items in the Test Booklet, you have to
fill in some particulars in the Answer Sheet as per instructions sent to you with your Admission Certificate.
8. After you have completed filling in all your responses on the Answer Sheet and the examination has concluded,
you should hand over to the Invigilator only the Answer Sheet. You are permitted to take away with you the
Test Booklet.
9. Sheets for rough work are appended in the Test Booklet at the end.
10. Penalty for wrong answers:
THERE WILL BE PENALTY FOR WRONG ANSWERS MARKED BY A CANDIDATE IN THE OBJECTIVE
TYPE QUESTION PAPERS.
(i) There are four alternatives for the answer to every question. For each question for which a wrong answer
has been given by the candidate, one-third of the marks assigned to that question will be deducted as
penalty.
(ii) If a candidate gives more than one answer, it will be treated as a wrong answer even if one of the given
answers happens to be correct and there will be same penalty as above to that question.
(iii) If a question is left blank, i.e., no answer is given by the candidate, there will be no penalty for that
O

question.
C
S.

DO NOT OPEN THIS TEST BOOKLET UNTIL YOU ARE ASKED TO DO SO


TE
O
FN
D
.P
W
W
W
https://www.pdfnotes.co/

1. Consider the following statements regarding Which of the statements given above is/are
the United Nations Office for Outer Space correct?
Affairs (UNOOSA): (a) Only one
1. UNOOSA contributes to raise global (b) Only two
adherence for legal instruments
applicable to outer space activities. (c) All three
(d) None
2. The United Nations office is responsible
to implement the United Nations
Programme on Space Applications 4. Regarding the ‘short-lived halogens’, consider
the following statements:
(PSA).
1. They have a lifetime of less than six
3. It is headquartered in California, USA.
months in the atmosphere.
Which of the statements given above is/are 2. They are naturally produced by the
correct? oceans.
(a) Only one 3. They decrease methane’s lifetime in
the atmosphere by destroying hydroxyl

RE
(b) Only two radicals (OH).
(c) All three How many of the statements given above are
(d) None correct?
(a) Only one
2. Consider the following statements regarding (b) Only two
the Strategic Intervention for Green Hydrogen
Transition (SIGHT):
O (c) All three
(d) None
1. It aims to establish green hydrogen
producing facilities only.
SC
5. Consider the following statements regarding
2. The program is a financial measure under the Programme for Restoration, Awareness,
the National Green Hydrogen Mission. Nourishment and Amelioration of Mother
Earth’ (PRANAM):
3. Solar Energy Corporation of India
(SECI) is the implementing agency for 1. The scheme is to incentivise States and
implementation of this scheme. Union Territories to promote alternative
fertilisers and counteracted use of
How many of the statements given above are chemical fertilisers.
GS

correct? 2. This scheme aims to reduce in consumption


(a) Only one of chemical fertilizers namely for Urea,
DAP, NPK, MOP.
(b) Only two
Which of the statements given above is/are
(c) All three correct?

(d) None (a) 1 only


(b) 2 only
3. With reference to the Panchayat Development
(c) Both 1 and 2
Index (PDI), consider the following statements:
(d) Neither 1 nor 2
1. It is a statistical tool to measure
the implementation of Sustainable
6. Consider the following:
Development Goals (SDGs).
1. Rotting of the flesh
2. It has Women, Child and Education as
major indicators. 2. High blood pressure

3. It was formulated by the respective state 3. Increases heart rate


O

governments. 4. Difficulty in breathing


C
S.
TE

2
O

PPP-PTS-4392/082023/17
FN
D
.P
W
W
W
https://www.pdfnotes.co/

How many of the above are caused by Zombie 10. Which of the following is the correct objective
drug xylazine? of Elucid mission, recently seen in news?

(a) Only one (a) To create the largest, most accurate 3D


map of the universe.
(b) Only two (b) To image the Sun and huge swaths of its
(c) Only three corona.
(c) To explore the planet Mercury.
(d) All four
(d) To map Moon’s Southern Pole.
7. The Ol Chiki script, recently mentioned in
news, was created for- 11. Consider the following statements:
1. Deep sea mining involves removing
(a) Mon-Khmer mineral deposits and metals from the
(b) Sarada ocean’s seabed.
2. Stripping cobalt crusts from rock is
(c) Ho considered as a type of deep sea mining.
(d) Santhali

RE
3. Rules related to Deep sea mining are
applied to countries that ratify the UN
8. Who is the Solicitor General of India (SGI) and Convention on law of the seas.
what is their role? How many of the above statements are
correct?
(a) The Solicitor General is the chief legal
advisor to the President of India.O (a) Only one
(b) Only two
(b) The Solicitor General is the chief legal
advisor to the Prime Minister of India. (c) All three

(c) The Solicitor General is the chief legal (d) None


SC
advisor to the Government of India.
12. Regarding the Santhal Hul (revolution),
(d) The Solicitor General is the chief legal consider the following statements:
advisor to the Supreme Court of India.
1. It led against the Santhal Pargana
Tenancy Act, which outlawed the transfer
9. With reference to Cooperative Societies, of Adivasi land to non-Adivasis.
consider the following statements: 2. The rebellion came to an end after 1857.
GS

1. Co-operative societies are profit-oriented 3. The movement was led in the eastern
entities. Bihar, including parts of West Bengal.

2. These societies work on the principle of How many of the statements given above are
correct?
self-help as well as mutual help.
(a) Only one
3. The Multi-State Co-operative Societies
Act of 2002 governs all the societies who (b) Only two
aim to serve the interests of its members (c) All three
in a particular State or in more than one
(d) None
state.

How many of the statements given above are 13. Regarding the International Seabed Authority
correct? (ISA), consider the following statements:
1. ISA has the mandate to ensure the
(a) Only one
effective protection of the marine
(b) Only two environment from harmful effects that
may arise from deep-seabed-related
(c) All three activities.
O

2. India is not a member of ISA.


C

(d) None
S.
TE

3
O

PPP-PTS-4392/082023/17
FN
D
.P
W
W
W
https://www.pdfnotes.co/

Which of the statements given above are How many of the statements given above are
correct? correct?
(a) 1 only (a) Only one
(b) 2 only (b) Only two
(c) Both 1 and 2 (c) All three
(d) Neither 1 nor 2 (d) None

14. Consider the following statements regarding 17. Consider the following statements regarding
Soil microbiomes: Bharat 6G Alliance (B6GA):
1. The soil microbiome governs 1. It is a collaborative platform consisting
biogeochemical cycling of macronutrients, of public and private companies and
micronutrients and other elements vital standards development organizations.
for the growth of plants.
2. It involves deployment of 6G technologies
2. Soil microbiomes work as a reservoir of to act as a powerful force multiplier for
human pathogens that induce antibiotic India by 2030.
resistance genes (ARGs)

RE
Which of the statements given above is/are
3. They directly influence the quality of the correct?
environment.
(a) 1 only
4. They provide a habitat for
microorganisms. (b) 2 only
How many of the statements given above are (c) Both 1 and 2
correct?
O (d) Neither 1 nor 2
(a) Only one
(b) Only two 18. Which of the following is the correct objective
of ‘Data Scraping’?
SC
(c) Only three
(a) Importing information from a website
(d) All four
(b) Clearing site data from a particular site
15. Consider the following: (c) Removing information that is personal,
1. Bangladesh irrelevant, or harmful

2. Maldives (d) All of the above


GS

3. India
19. Regarding the National Tele Mental Health
4. Sri Lanka Programme of India, Tele Mental Health
Assistance and Networking across States (Tele-
How many of the above countries are members
MANAS), consider the following statements:
of the Colombo Security Conclave (CSC)?
1. Tele-MANAS provide free tele-mental
(a) Only one
health services in two-tier system.
(b) Only two
2. All District Hospitals in States are
(c) Only three included under the network.
(d) All four 3. The counsellors are being appointed
under the State Tele MANAS Cell.
16. Consider the following statements regarding How many of the above statements are
Quasar: correct?
1. It is a supermassive black hole feeding on (a) Only one
gas at the center of a distant galaxy.
(b) Only two
2. Quasars are a subclass of active galactic
nuclei (AGNs). (c) All three
O

(d) None
C

3. All AGNs are quasars.


S.
TE

4
O

PPP-PTS-4392/082023/17
FN
D
.P
W
W
W
https://www.pdfnotes.co/

20. Consider the following statements regarding 24. Consider the following statements regarding
the Indian gazelle (Gazella bennettii): African Union (AU):
1. It can be found in open scrub, thorn 1. The organisation aims to promote and
forests, and open dry deciduous forest. accelerate the political and socio-economic
2. In India, the largest population of the integration of the African continent.
Chinkara can be found in Meghalaya.
2. African Union was proposed as the full-
3. They are endangered under IUCN Red time member of G20 by India during 20
List. Summit in New Delhi.
How many of the statements given above are
correct? Which of the statements given above is/are
correct?
(a) Only one
(a) 1 only
(b) Only two
(b) 2 only
(c) All three
(c) Both 1 and 2
(d) None
(d) Neither 1 nor 2

RE
21. Consider the following particles:
1. Sulfur dioxide 25. Consider the following statements:
2. Calcium carbonate 1. Cluster Bombs are canisters that contain
tens to hundreds of smaller bomblets.
3. Diamond dust
4. Calcium Carbonate 2. The canisters can be launched using
missiles, or fired from naval guns,

pumped into the upper atmosphere through


O
How many of the above given particles can be

the process of ‘stratospheric aerosol injection’ 3.


artillery, or rocket launchers.
Using cluster bombs is a war crime.
to cool down the earth?
How many of the statements given above is/
SC
(a) Only one are correct?
(b) Only two
(a) Only one
(c) Only three
(b) Only two
(d) All four
(c) All three
22. Which of the following is correct regarding (d) None
GS

‘Extractive Economy’?
(a) The primary focus of such economies is 26. With reference to ‘Western Disturbance (WD)’,
on exploiting natural resources. consider the following statements:
(b) In such economies, the government 1. The disturbance travels from the
has significant control over resource
“western” to the eastern direction.
allocation, production, and distribution.
(c) It places a greater focus on innovation, 2. Disturbance is an area of high air
research and development, and pressure.
intellectual capital.
3. The extra tropical storm is associated
(d) All of the above with rainfall, snowfall and fog.
How many of the statements given above is/
23. The Tomato leaf curl virus (ToLCV) is
are correct?
transmitted by-
(a) Thrips (a) Only one

(b) Leafhoppers (b) Only two


(c) Aphids (c) All three
O

(d) Whiteflies
C

(d) None
S.
TE

5
O

PPP-PTS-4392/082023/17
FN
D
.P
W
W
W
https://www.pdfnotes.co/

27. Consider the following statements regarding 30. Regarding the National Securities Depository
Global Crisis Response Group (GCRG): Ltd. (NSDL), consider the following
statements:
1. It aims to address critical global issues
1. NSDL aims at ensuring the safety and
pertaining to interlinked crises in food soundness of Indian marketplaces by
security, energy, and finance. developing settlement solutions
2. It was set up by the UN Secretary- 2. It introduced the concept of
General in 2022. dematerialization of securities in India.

3. It has 32-members including Bangladesh, 3. NSDL is the depository operating in


India.
Barbados, Denmark, Germany and
Indonesia. How many of the statements given above is/
are correct?
4. India is not a member to the group.
(a) Only one
How many of the statements given above is/ (b) Only two
are correct?
(c) All three
(a) Only one

RE
(d) None
(b) Only two
31. ‘Phosphine’, recently mentioned in news, is
(c) Only three
produced by-
(d) All four (a) Heating either natural gas or petroleum.
(b) From the monomer styrene.
O
28. Seethakali, a traditional folk art form, has its
(c) Heating a mixture of chlorine and either
roots in which region of India?
methyl chloride
(a) Western Ghats of Karnataka
(d) Tiny bacteria that live in very low-oxygen
environments.
SC
(b) Northern Himalayan region

(c) Eastern coastal areas of Odisha 32. Consider the following statements regarding
the Small Satellite Launch Vehicle (SSLV):
(d) Central plains of Madhya Pradesh
1. The SSLV provides on-demand services
for launching satellites weighing up to
29. Consider the following statements regarding
500 kg into a low-earth orbit.
Guillain-Barré Syndrome:
GS

2. It facilitates flexibility in accommodating


1. It is a rare neurological disease where multiple satellites.
a person’s immune system attacks their
Which of the statements given above is/are
body’s peripheral nerves. correct?
2. In GBS, the myelin sheath becomes (a) 1 only
inflamed and can hardly carry stimuli.
(b) 2 only
3. Zika virus infection is a trigger of (c) Both 1 and 2
Guillain-Barré syndrome.
(d) Neither 1 nor 2
4. The disease is more common in children.

How many of the statements given above is/ 33. Regarding Brucellosis, consider the following
statements:
are correct?
1. It is a viral disease which mainly affects
(a) Only one cattle.
(b) Only two 2. It typically causes flu-like symptoms,
including fever, weakness, malaise and
(c) Only three weight loss.
O

(d) All four 3. Person-to-person transmission is rare.


C
S.
TE

6
O

PPP-PTS-4392/082023/17
FN
D
.P
W
W
W
https://www.pdfnotes.co/

Which of the statements given above is/are 36. Consider the following statements regarding
correct? the Association of World Election Bodies
(A-WEB):
(a) Only one
1. It is an association of Election
(b) Only two Management Bodies (EMBs).
(c) All three 2. The permanent secretariat of A-WEB is
located in Seoul, South Korea.
(d) None
3. India is the member of A-WEB.
34. Consider the following statements about Shelf Which of the statement(s) given above is/are
Cloud: correct?

1. They are often associated with powerful (a) Only one


storm systems. (b) Only two
2. They are also known as Arcus clouds. (c) All three

3. It is much smaller than a wall cloud. (d) None

RE
4. A shelf cloud may appear to rotate on a
37. The ‘AuditOnline’ application, recently
horizontal axis. mentioned in news, is an initiative of-
How many of the statements given above are (a) Ministry of Statistics and Programme
correct? Implementation
(a) Only one (b) Ministry of Labour and Employment

(b)

(c)
Only two

Only three
O (c) Ministry of
Development
Women and Child

(d) Ministry of Panchayati Raj


SC
(d) All four
38. With reference to the grouping of ‘European
35. Consider the following statements regarding Sky Shield Initiative’ (ESSI), consider the
the ‘Differential Global Navigation Satellite following statements:
System (DGNSS)’ called Sagar Sampark.: 1. The European Sky Shield Initiative
1. It is a terrestrial based enhancement (ESSI) seeks to develop a common air
and missile defence system to defend
GS

system.
NATO and European airspace.
2. The Global Navigation Satellite 2. All NATO members are part of the
System (GNSS) provides more accurate initiative.
positioning information than DGNSS.
Which of the statements given above is/are
3. The made-in-India navigation system correct?
helps mariners to improve their (a) 1 only
positioning within 5 meters after
recapitalisation with multiple satellite (b) 2 only
constellations. (c) Both 1 and 2
How many of the statements given above are (d) Neither 1 nor 2
correct?
39. With reference to the Atal Vayo Abhyuday
(a) Only one
Yojana (AVYAY), consider the following
(b) Only two statements:

(c) All three 1. It is a comprehensive initiative aimed at


empowering senior citizens and widows
O

(d) None
C

in India.
S.
TE

7
O

PPP-PTS-4392/082023/17
FN
D
.P
W
W
W
https://www.pdfnotes.co/

2. The financial assistance is provided to 42. The ‘Right to Silence’, emanates from which
eligible organizations for running and of the following Article of the Constitution of
maintenance of Senior Citizen Homes/ India:
Continuous Care Homes.
(a) Article19(1)(a)
Which of the statements given above is/are
correct? (b) Article 19(1)(b)

(a) 1 only (c) Article 20(3)


(b) 2 only (d) None of these
(c) Both 1 and 2
43. Consider the following statements regarding
(d) Neither 1 nor 2 the ‘Gulf Stream’, recently mentioned in
news:
40. Consider the following statements regarding
the International Organization for Migration 1. The Gulf Stream is a warm and swift
(IOM): Atlantic Ocean current that follows the
eastern coastline of the US and Canada
1. IOM works to facilitate, regulate and before crossing the Atlantic Ocean

RE
address forced migration. towards Europe.
2. IOM was accorded the status of a
2. It ensures that the climate of Western
Permanent Observer to the United
Europe is much warmer than it would
Nations General Assembly.
otherwise be.
3. It is headquartered in Geneva,
Switzerland. Which of the statements given above is/are
correct?
O
How many of the statements given above are
correct? (a) 1 only

(a) Only one (b) 2 only


SC
(b) Only two (c) Both 1 and 2
(c) All three (d) Neither 1 nor 2
(d) None
44. Consider the following statements regarding
the Agriculture Infra Fund (AIF):
41. With reference to ISRO’s Chandrayaan-3
mission, consider the following statements: 1. It is a medium-long term debt financing
GS

1. Chandrayaan-3 mission consists of an for investment in viable projects for post-


indigenous propulsion module, lander harvest management infrastructure.
module and a rover. 2. It is a centrally sponsored scheme.
2. Similar to Chandrayaan-2,
Which of the statements given above is/are
Chandrayaan-3 carries an instrument
correct?
named the Spectro-polarimetry of
HAbitable Planet Earth (SHAPE). (a) 1 only
3. Its lander includes Chandra’s Surface (b) 2 only
Thermophysical Experiment (ChaSTE)
to measure thermal conductivity and (c) Both 1 and 2
temperature on the surface.
(d) Neither 1 nor 2
How many of the statements given above are
correct? 45. Consider the following statements regarding
(a) Only one the Mission Organic Value Chain Development
for North Eastern Region:
(b) Only two
1. The scheme aims at development of
(c) All three
certified organic production in a value
O

(d) None chain mode.


C
S.
TE

8
O

PPP-PTS-4392/082023/17
FN
D
.P
W
W
W
https://www.pdfnotes.co/

2. The National Advisory Committee is 49. With reference to Export Preparedness Index
the overall policy making body for the (EPI), consider the following statements:
mission.
1. It is a comprehensive tool to measure the
3. Six North-eastern states have been export preparedness of the States and
selected for the mission. UTs in India.

How many of the statements given above is/ 2. EPI assesses the performance across four
are incorrect? pillars – Policy, Business Ecosystem,
Export Ecosystem, and Export
(a) Only one Performance.
(b) Only two 3. This annual publication is released by
the Ministry of Commerce and Industry.
(c) All three
How many of the statements given above are
(d) None
correct?

46. Which of the following agricultural practices (a) Only one


involves the intentional integration of trees, (b) Only two
forage crops, and livestock on the same land

RE
area, aiming to create a sustainable and (c) All three
productive system? (d) None
(a) Pomology
50. Which of the following correctly define
(b) Horticulture ‘Goldilocks scenario’ in an economy?
(c) Silvopasture O (a) It refers to an ideal situation where there
(d) Arboriculture is steady growth.
(b) It is a condition where interest rates
47. Which of the following vector-borne diseases are extremely low, hindering economic
is caused by the bite of infected black-legged growth.
SC
ticks and is characterized by symptoms such as
(c) It is a scenario characterized by extreme
fever, fatigue, headache, and a characteristic income inequality, impacting overall
“bull’s-eye” rash? economic health.
(a) Lyme disease (d) It is the equilibrium condition in which
(b) Chikungunya long term economic forecasts can be
made.
(c) Japanese encephalitis
GS

(d) Yellow fever 51. Consider the following statements with


reference to Indian National Space Promotion
and Authorisation Centre (IN-SPACe):
48. Consider the following statements regarding
Electric Freight Accelerator for Sustainable 1. It is a single-window, independent, nodal
Transport – India, or e-FAST: agency that functions as an autonomous
agency in the Department of Space
1. It is India’s first national electric freight (DOS).
platform.
2. The agency acts as an interface between
2. The platform is aimed at accelerating ISRO and Non-Governmental Entities
freight electrification in India. (NGEs).
Which of the statements given above is/are Which of the statements given above is/are
correct? correct?
(a) 1 only (a) 1 only
(b) 2 only (b) 2 only
(c) Both 1 and 2 (c) Both 1 and 2
O

(d) Neither 1 nor 2


C

(d) Neither 1 nor 2


S.
TE

9
O

PPP-PTS-4392/082023/17
FN
D
.P
W
W
W
https://www.pdfnotes.co/

52. Which of the following is the correct objective How many of the statements given above is/
of the G20 EMPOWER initiative launched are incorrect?
under India’s presidency in 2023?
(a) Only one
(a) To promote digital literacy among women
(b) Only two
and children in India’s rural areas.
(c) All three
(b) To enhance women’s leadership and
empowerment in the private sector (d) None
through collaboration between G20
business leaders and governments. 55. Consider the following statements regarding
Trachoma:
(c) To provide financial assistance to women
entrepreneurs in developing countries 1. It is an eye disease caused by infection
for launching tech-based startups. with the bacterium Chlamydia
trachomatis.
(d) To facilitate international trade
agreements that promote gender equality 2. The infection is transmitted by direct
or indirect transfer of eye and nose
and women’s economic representation.
discharges of infected people.

RE
53. Consider the following statements regarding 3. Blindness from trachoma is reversible.
the ‘Finger Minutiae Record – Finger Image How many of the statements given above are
Record (FMR-FIR) modality’, recently seen in correct?
news:
(a) Only one
1. It is an Artificial Intelligence/Machine (b) Only two
Learning based tool.
O
(c) All three
2. It can check the liveness of a fingerprint
to detect the use of a cloned fingerprint (d) None
during the authentication process to
SC
prevent AePS frauds. 56. Consider the following statements about
Himalayan vulture (Gyps himalayensis):
3. It is developed by the National Informatics
Centre (NIC). 1. It is the largest of the Gyps species.
2. It is native to the Himalayas and the
How many of the statements given above are
adjoining Tibetan Plateau.
correct?
3. The Himalayan Griffon Vulture feeds
GS

(a) Only one only on carrion.


(b) Only two How many of the statements given above are
correct?
(c) All three
(a) Only one
(d) None
(b) Only two
54. Regarding Infrastructure Investment Trust (c) All three
(InvITs), consider the following statements:
(d) None
1. It enables direct investment of small
amounts of money from possible 57. Consider the following statements regarding
individual/institutional investors in ‘Yellow Belly Snake’:
infrastructure to earn a small portion of
1. The species is found in tropical and
the income as return. subtropical waters around the world.
2. They are like mutual funds in structure. 2. It is a diurnal sea snake and primarily
3. They are regulated by the SEBI aquatic.
(Infrastructure Investment Trusts) 3. This highly venomous species of snake is
O

Regulations, 2014. Critically Endangered.


C
S.
TE

10
O

PPP-PTS-4392/082023/17
FN
D
.P
W
W
W
https://www.pdfnotes.co/

How many of the statements given above are 61. The Committee of Parliament on Official
correct? Language is provided under which of the
following Article of Indian Constitution?
(a) Only one
(a) Article 29
(b) Only two
(b) Article 250
(c) All three
(c) Article 344
(d) None
(d) None of these
58. Which of the following is the primary objective
of the Lunar Codex initiative launched during 62. Consider the following statements regarding
NASA’s Artemis- I mission? the Bhu-Vision (Krishi-RASTAA):
(a) To establish a colony of artists on the Statement-I:
moon and encourage creativity in space.
Its primary purpose is to enable soil testing
(b) To remind future generations that war, and analysis, helping farmers determine the
pandemics, and economic crises did not health and nutrient content of their soil.

RE
deter artistic endeavors.
Statement-II:
(c) To study the effect of lunar conditions on
various forms of art and communication. It is developed by Indian Council of Agricultural
Research (ICAR).
(d) None of these
Which one of the following is correct in respect
of the above statements?
the Parliamentary Privileges:
1.
O
59. Consider the following statements regarding

Article 105 and Article 194 grant privileges


(a) Both Statement-I and Statement-II are
correct and Statement-II is the correct
explanation for Statement-I
to the members of the parliament.
SC
(b) Both Statement-I and Statement-II
2. The privileges are not absolute in
are correct and Statement-II is not the
nature.
correct explanation for Statement-I
Which of the statements given above is/are
(c) Statement-I is correct but Statement-II
correct?
is incorrect
(a) 1 only
GS

(d) Statement-I is incorrect but Statement-


(b) 2 only II is correct
(c) Both 1 and 2
63. Consider the following statements regarding
(d) Neither 1 nor 2 Spike non-line of sight (NLOS) long range
anti-tank missile:
60. ‘Einstein Cross’, a prominent astronomical
phenomenon, is associated with: 1. It is a family of anti-tank missiles made
by Rafael Advanced Defense Systems.
(a) The discovery of a new element in the
periodic table. 2. It only relies on fibre-optic technology.

(b) The splitting of light into its component Which of the statements given above is/are
colors using a prism. correct?

(c) The multiple images of a distant quasar (a) 1 only


formed due to gravitational lensing by an (b) 2 only
intervening galaxy.
(c) Both 1 and 2
(d) The occurrence of solar eclipses in a
O

regular pattern. (d) Neither 1 nor 2


C
S.
TE

11
O

PPP-PTS-4392/082023/17
FN
D
.P
W
W
W
https://www.pdfnotes.co/

64. With respect to the Central Registrar of How many of the statements given above are
Cooperative Societies, consider the following correct?
statements: (a) Only one
1. The Central Registrar of Cooperative (b) Only two
Societies is appointed as per article
243ZH(f) of the Constitution. (c) All three

2. It is the statutory body responsible (d) None


for registration and other processes of
the Multi State Cooperative Societies 67. Regarding the ‘compulsory convertible
(MSCS). debenture (CCD)’, consider the following
statements:
3. The Cooperative societies with objects
confined to one State only are governed 1. It is a type of bond which must be
by the Multi-State Co-operative Societies converted into stock by a specified date.
Act 2002.
2. For companies, it does not allow for
How many of the statements given above are repayment of debt without spending
correct? cash.

RE
(a) Only one 3. It is purely a bond.
(b) Only two How many of the statements given above are
correct?
(c) All three
(a) Only one
(d) None
(b) Only two
O
65. Regarding the Digital India RISC-V (DIR-V) (c) All three
Program, consider the following statements:
(d) None
1. The DIR-V program intends to make India
SC
self-reliant in semiconductor technology
68. Consider the following statements with
2. This program is a component of the reference to the Chief Election Commissioner
Semicon India program. (CEC):
3. RISC-V is an open-source instruction 1. The Chief Election Commissioner can
set architecture (ISA) that enables the only be expelled by 2/3rd of the members of
development of custom processors for Lok Sabha and Rajya Sabha combined.
GS

various end applications.


2. India’s CEC serves a term of 6 years or
How many of the statements given above are 65 years, whichever comes first.
correct?
3. He handles the complete voting procedure
(a) Only one on the day of polling.
(b) Only two How many of the statements given above are
correct?
(c) All three
(a) Only one
(d) None
(b) Only two
66. Consider the following statements regarding (c) All three
Software-as-a-Service (SaaS):
(d) None
1. It is a cloud based software delivery
model.
69. Consider the following statements regarding
2. With a SaaS model, the software is hosted Cloudburst:
on a local server.
1. Cloudburst is a sudden, very heavy
3. SaaS provides Multi-Tenant rainfall, usually local in nature and of
O

brief duration.
C

Architecture.
S.
TE

12
O

PPP-PTS-4392/082023/17
FN
D
.P
W
W
W
https://www.pdfnotes.co/

2. Most cloudbursts occur in connection 72. Consider the following statements with
with thunderstorms. reference to the ‘Community Reserves’ in
India:
3. It occurs due to the formation of high-
pressures areas in high-altitude regions. 1. They fall under protected areas.

How many of the statements given above are 2. They typically act as buffer zones
between established national parks,
correct?
wildlife sanctuaries and reserved and
(a) Only one protected forests of India.

(b) Only two 3. Any community or private land may


be designated directly by the Centre
(c) All three as a Community Reserve provided the
(d) None inhabitants of that community or the
person concerned consent to provide such
areas for the preservation
70. Regarding Deflation, consider the following
statements: How many of the statements given above are
correct?

RE
Statement-I:
(a) Only one
It is a fall in the overall level of prices in an
(b) Only two
economy and an increase in the purchasing
power of the currency. (c) All three

Statement-II: (d) None

economy.
O
It is only caused due to decreased productivity
or a decrease in the volume of credit in the
73. With reference to IMF Quotas, consider the
following statements:
1. The IMF Quotas are the building blocks
Which one of the following is correct in respect
SC
of the IMF’s financial and governance
of the above statements? structure.
(a) Both Statement-I and Statement-II are 2. A quota formula is used to help assess
correct and Statement-II is the correct members’ relative position in the world
explanation for Statement-I economy.

(b) Both Statement-I and Statement-II 3. Quotas are denominated in dollars.


are correct and Statement-II is not the
GS

How many of the statements given above are


correct explanation for Statement-I correct?
(c) Statement-I is correct but Statement-II (a) Only one
is incorrect
(b) Only two
(d) Statement-I is incorrect but Statement-
(c) All three
II is correct
(d) None
71. Which of the following immortal cell line,
derived from cervical cancer cells in 1951, 74. Consider the following statements with
has played a pivotal role in medical research, reference to the Inclusive Conservation
including vaccine development and cancer Initiative (ICI):
studies? 1. Its goal is to enhance Indigenous
Peoples’ and Local Communities’ (IPs
(a) ME-180 cells
and LCs) efforts to steward land, waters
(b) A549 cells and natural resources to deliver global
environmental benefits.
(c) HeLa cells
2. It is funded by the Global Environment
O

(d) WI-38 cells Facility (GEF).


C
S.
TE

13
O

PPP-PTS-4392/082023/17
FN
D
.P
W
W
W
https://www.pdfnotes.co/

Which of the statements given above is/are 2. Battens Disease is passed down from
correct? parents to children.
(a) 1 only 3. The disease is curable.
(b) 2 only How many of the statements given above are
correct?
(c) Both 1 and 2
(a) Only one
(d) Neither 1 nor 2
(b) Only two
75. According to the provisions of the Central (c) All three
Goods & Services Tax (CGST) Act, 2017, the
Consumer Welfare Fund (CWF) is primarily (d) None
utilized for-
78. “Despite being a minor, she managed to join the
(a) Providing financial assistance to
Mrityu Bahini group. In 1942, she led a group
manufacturers affected by GST
of 5,000 freedom fighters on a march towards
regulations
Gohpur Police station to hoist the Tricolour in
(b) Compensating consumers for any losses support of the Quit India Movement. Police at

RE
incurred due to changes in GST rates the station opened indiscriminate fire on the
group to prevent the team from committing
(c) Supporting government initiatives to
such a daring act. She was martyred at a young
enhance consumer awareness about
age with a Tricolour furling in her hands.”
GST
The above para is referred to-
(d) Funding research and development
projects for improving GST (a) Mukunda Kakoty
infrastructure
O (b) Kanaklata Barua

76. With reference to legionella bacteria, consider (c) Rani Gaidinliu


the following statements: (d) Santi Ghose
SC
1. Legionella bacteria proliferate in water
systems with appropriate temperature 79. Consider the following statements regarding
conditions, such as hot tubs and air the Pradhan Mantri Uchchatar Shiksha
conditioners. Abhiyan (PM-USHA):
2. Legionnaires’ disease, caused by inhaling 1. It is a centrally sponsored programme.
small droplets of water containing the
bacteria, can lead to severe pneumonia. 2. The scheme aims at providing strategic
GS

funding to eligible state higher


3. Pontiac fever, a more severe illness than educational institutions.
legionnaires’ disease, is the common
outcome when exposed to legionella 3. The central funding is based on norms
bacteria. and is outcome dependent.

How many of the statements given above are How many of the statements given above are
correct? correct?

(a) Only one (a) Only one

(b) Only two (b) Only two

(c) Only three (c) All three

(d) None (d) None

77. Consider the following statements regarding 80. Regarding the National Automated
Type 2 Battens Disease: Fingerprints Identification System (NAFIS),
consider the following statements:
1. It is a group of genetic disorders classified
as neuronal ceroid lipofuscinoses 1. It is a country-wide searchable database of
O

crime- and criminal-related fingerprints.


C

(NCLs).
S.
TE

14
O

PPP-PTS-4392/082023/17
FN
D
.P
W
W
W
https://www.pdfnotes.co/

2. It functions as a central information 83. Regarding the Promotion of Research &


repository by consolidating fingerprint Innovation in Pharma-MedTech (PRIP),
data from all states and Union consider the following statements:
Territories. 1. The objective of the scheme is to transform
3. It assigns 16-digit National Fingerprint the Indian Pharma MedTech sector from
Number (NFN) to different sets of cost-based to innovation-based growth.
crimes. 2. The scheme provides for the setting of
4. It is managed by the NCRB. Centres of Excellence at the National
Institute of Pharmaceutical Education &
Which of the statements given above is/are Research (NIPERs).
correct?
Which of the statements given above is/are
(a) Only one correct?

(b) Only two (a) 1 only

(c) Only three (b) 2 only

(d) All four (c) Both 1 and 2


(d) Neither 1 nor 2

RE
81. With reference to ‘metagenome sequencing’,
consider the following statements: 84. The joint statement “The Spirit of Camp
David”, is adopted by-
1. It is the direct genetic analysis of
genomes (a) United States, Japan and South Korea
2. It allows researchers to comprehensively (b) United States, Japan and Philippines
sample all genes in all organisms present
(c) Japan, South Korea and Maldives
in a given complex sample.
3. It utilizes only 16S rRNA analysis.
How many of the statements given above are
O (d) Japan and Philippines

85. Consider the following statements regarding


correct? the Infrastructure Debt Funds - Non-Banking
SC
(a) Only one Financial Companies (IDF-NBFCs):
1. An IDF-NBFC is a non-deposit-taking
(b) Only two
NBFC that is authorised to refinance
(c) Only three infrastructure projects that have
completed at least one year of satisfactory
(d) None commercial operations.
2. IDF-NBFCs are not allowed to raise funds
82. Consider the following statements with through rupee or dollar-denominated
GS

reference to Maharishi Dayanand Saraswati: bonds.


1. He created awareness in the context of 3. They are subject to exposure limits.
cow protection, Hindi protection, utility of
Sanskrit language, Swadesh, Swadeshi, How many of the statements given above are
correct?
Swabhasha, and self-respect.
(a) Only one
2. He established Arya Samaj in Mumbai
in 1875 for continuous expansion of his (b) Only two
work.
(c) All three
3. He wrote a book Hindu-Sangathan
(d) None
that played a key role in the revival of
Hinduism.
86. Consider the following statements regarding
How many of the statements given above are Kashmir stag (Cervus hanglu hanglu):
correct?
1. It is a subspecies of Central Asian red
(a) Only one deer only found in Kashmir.
(b) Only two 2. This critically-endangered species lives
(c) All three in groups in dense riverine forests
O

3. It is the only red deer species in India.


C

(d) None
S.
TE

15
O

PPP-PTS-4392/082023/17
FN
D
.P
W
W
W
https://www.pdfnotes.co/

How many of the statements given above are How many of the statements given above are
correct? correct?
(a) Only one (a) Only one

(b) Only two (b) Only two

(c) All three (c) All three

(d) None (d) None

87. Consider the following statements regarding 90. Which of the following is NOT among the
Regions of the Commonwealth Parliamentary
ASEAN-India Trade in Goods Agreement
Association (CPA)?
(AITIGA):
1. It aims to promote knowledge of the
1. It is a free trade agreement (FTA) constitutional, legislative, economic, social
between the Association of Southeast and cultural aspects of parliamentary
Asian Nations (ASEAN) and India. democracy.
2. AITIGA aims to reduce or eliminate 2. It is a membership association.
tariffs on a wide range of goods traded

RE
between ASEAN member states and 3. India is not among the regions of the
CPA.
India.
How many of the statements given above are
How many of the statements given above are
correct?
correct?
(a) Only one
(a) 1 only
(b) 2 only
O (b) Only two
(c) All three
(c) Both 1 and 2
(d) None
(d) Neither 1 nor 2
SC
91. Regarding the ‘Astra’ Missile, consider the
88. Which of the following is the main reason that following statements:
makes landing a spacecraft on the Moon’s
1. It is Beyond Visual Range Air-to-Air
South Pole challenging?
Missile (BVRAAM).
(a) The Moon’s gravitational pull is stronger
2. It is designed and developed by the
at its south pole. Defence Research and Development
GS

(b) Solar radiation is more intense at the Organisation.


Moon’s South Pole. 3. It is developed for deployment on fighter
(c) The South Pole region has a difficult jets like Sukhoi-30 MKI and Tejas of the
IAF and the Mig-29K of the Navy.
terrain, full of craters and deep
trenches. How many of the statements given above are
correct?
(d) None of these
(a) Only one
89. Consider the following statements with (b) Only two
reference to Bharat NCAP:
(c) All three
1. The safety assessment programme is to
(d) None
be applicable to all passenger vehicles
that can seat up to eight people and
weigh less than 3.5 tonnes. 92. Consider the following statements regarding
the Global Antibiotic Research and
2. The Bharat NCAP is aligned with global Development Partnership (GARDP):
crash test protocols.
1. It is a not-for-profit organization
3. Similar to Global NCAP, Bharat NCAP developing new treatments for drug-
O

will just comprise a standardized test. resistant infections.


C
S.
TE

16
O

PPP-PTS-4392/082023/17
FN
D
.P
W
W
W
https://www.pdfnotes.co/

2. It was established by the United Nations 2. It is designed to mobilize and accelerate


Office for Disaster Risk Reduction. investment in the conservation and
sustainability of wild species and
Which of the statements given above is/are
ecosystems.
correct?
(a) 1 only 3. It will also prioritise support for Small
Island Developing States and Least
(b) 2 only Developed Countries.
(c) Both 1 and 2 How many of the statements given above are
(d) Neither 1 nor 2 correct?
(a) Only one
93. Which of the following correctly defines
(b) Only two
‘Mylara’, often mentioned in news?
(c) All three
(a) A movement advocating for environmental
conservation in urban areas. (d) None
(b) A folk deity identified as a manifestation

RE
of Lord Shiva. 96. With reference to the Northern Sea Route
(NSR), often mentioned in news, consider the
(c) A tribal community focused on promoting
following statements:
traditional agricultural practices.
1. It connects the eastern and western parts
(d) A literary organization dedicated
of the Arctic Ocean.
to preserving local folklore and oral
traditions. 2. It runs along the Russian Arctic coast
O
94. Consider the following statements regarding
the Deep Space Network mission lunched by
from Kara Gates strait between the
Barents Sea and the Kara Sea, along
Siberia, to the Bering Strait.
NASA:
SC
3. The navigation distance via the Northern
1. It is an international network of huge Sea Route (NSR) from a Northwest-
antennas that allows people on the European port to the Far East is
ground to communicate with satellites approximately 60% longer compared to
and other spacecraft missions. the route via the Suez Canal.
2. It is the largest and most sensitive How many of the statements given above are
scientific telecommunications system in correct?
GS

the world.
(a) Only one
3. The DSN consists of a single facility that
permits constant communication with (b) Only two
spacecraft as the Earth rotates.
(c) All three
How many of the statements given above are
(d) None
correct?
(a) Only one 97. Regarding the ‘Bidri craft’, recently mentioned
(b) Only two in news, consider the following statements:

(c) All three 1. Bidri is created by a process of casting,


polishing, engraving, inlay, and
(d) None blackening alloy.

95. With reference to Global Biodiversity 2. The intricate brass work flourished during
Framework Fund (GBF-Fund), consider the the reign of the Kakatiya dynasty.
following statements: 3. The unique style was created through
1. It is a special trust fund under the Global the intermingling of Turkish, Persia and
O

Environment Facility (GEF). Arabic influences with the local styles.


C
S.
TE

17
O

PPP-PTS-4392/082023/17
FN
D
.P
W
W
W
https://www.pdfnotes.co/

How many of the statements given above are 1. It is maintained through a natural
correct? cycle between the Culiseta melanura
mosquitoes and birds.
(a) Only one
(b) Only two 2. The spread of EEEV is person-to-person,
people to animals, or animals to people.
(c) All three
3. There is no human vaccine for EEE.
(d) None
How many of the above statements given
above are correct?
98. Regarding the Gulf Cooperation Council,
consider the following statements: (a) Only one
1. It is a regional co-operation system (b) Only two
between the Arab States of the Gulf.
(c) All three
2. All current member states are
monarchies. (d) None

3. The similarity of GCC states’ economic


100. With reference to Raptors, consider the

RE
and social conditions were the sole factors
that helped in the establishment of the following statements:
GCC. 1. They are the apex predators in the bird
4. The Council is the third largest trading world
bloc of India. 2. They are distinguished by keen vision,
How many of the statements given above are powerful talons and beaks.
correct?
O 3. All Raptors are crepuscular.
(a) Only one
How many of the above statements are
(b) Only two correct?
SC
(c) Only three (a) Only one
(d) All four (b) Only two
(c) All three
99. With reference to Eastern equine encephalitis
(EEE), consider the following statements: (d) None
GS

™™™™™

O
C
S.
TE

18
O

PPP-PTS-4392/082023/17
FN
D
.P
W
W
W
https://www.pdfnotes.co/

space for rough work

RE
O
SC
GS

O
C
S.
TE

19
O

PPP-PTS-4392/082023/17
FN
D
.P
W
W
W
https://www.pdfnotes.co/

IAS Prelims - 2024


PPP-PTS-4392/082023/17

GENERAL STUDIES

CURRENT AFFAIRS-2
JULY - AUGUST 2023

Answer Key

Q. 1 (b) Q. 21 (d) Q. 41 (b) Q. 61 (c) Q. 81 (c)


Q. 2 (b) Q. 22 (a) Q. 42 (c) Q. 62 (a) Q. 82 (b)
Q. 3 (a) Q. 23 (d) Q. 43 (c) Q. 63 (a) Q. 83 (c)
Q. 4 (b) Q. 24 (c) Q. 44 (c) Q. 64 (b) Q. 84 (a)
Q. 5 (c) Q. 25 (b) Q. 45 (b) Q. 65 (c) Q. 85 (b)
Q. 6 (b) Q. 26 (b) Q. 46 (c) Q. 66 (b) Q. 86 (b)
Q. 7 (d) Q. 27 (b) Q. 47 (a) Q. 67 (a) Q. 87 (c)
Q. 8 (c) Q. 28 (a) Q. 48 (c) Q. 68 (c) Q. 88 (c)
Q. 9 (b) Q. 29 (c) Q. 49 (b) Q. 69 (b) Q. 89 (b)
Q. 10 (a) Q. 30 (c) Q. 50 (a) Q. 70 (d) Q. 90 (b)
Q. 11 (b) Q. 31 (d) Q. 51 (c) Q. 71 (c) Q. 91 (c)
Q. 12 (b) Q. 32 (c) Q. 52 (b) Q. 72 (b) Q. 92 (a)
Q. 13 (a) Q. 33 (b) Q. 53 (b) Q. 73 (b) Q. 93 (b)
Q. 14 (d) Q. 34 (c) Q. 54 (d) Q. 74 (c) Q. 94 (b)
Q. 15 (c) Q. 35 (b) Q. 55 (b) Q. 75 (c) Q. 95 (c)
Q. 16 (b) Q. 36 (c) Q. 56 (c) Q. 76 (b) Q. 96 (b)
Q. 17 (c) Q. 37 (d) Q. 57 (b) Q. 77 (b) Q. 97 (b)
Q. 18 (a) Q. 38 (a) Q. 58 (b) Q. 78 (b) Q. 98 (c)
Q. 19 (b) Q. 39 (c) Q. 59 (a) Q. 79 (c) Q. 99 (b)
Q. 20 (a) Q. 40 (c) Q. 60 (c) Q. 80 (c) Q. 100 (b)
O
C
S.
TE
O
FN
D
.P
W
W
W
https://www.pdfnotes.co/

1. Correct option: (b)  The programme proposes two distinct


financial incentive mechanisms to support
Explanation: domestic manufacturing of electrolysers
 Statement 3 is incorrect: Its headquarter and production of Green Hydrogen.
is in Vienna, Austria.  Project SIGHT has an outlay of Rs. 17,490
Supplementary notes crore which will be covered by the Rs 19,744
crore budget provisions for the National
United Nations Office for Outer Space Hydrogen Mission until FY30. Both schemes
Affairs (UNOOSA) will be implemented from FY26 to FY30 by
 The United Nations Office for Outer the Solar Energy Corporation of India
Space Affairs (UNOOSA) works to (SECI).
promote international cooperation in the
 The first component on electrolyser
peaceful use and exploration of space,
manufacturing has a budget of Rs.4, 440
and in the utilisation of space science and
crore while the second component on Green
technology for sustainable economic and
social development. Hydrogen has a total budget of Rs. 13,050
crore.
 The Office assists any United Nations
Member States to establish legal and Why in News?
regulatory frameworks to govern space  The incentives under the recently issued

RE
activities and strengthens the capacity of strategic interventions for green hydrogen
developing countries to use space science
transition (SIGHT) scheme will promote
technology and applications for development
the large-scale utilisation and enhance cost-
by helping to integrate space capabilities
effectiveness, as per the industry expert.
into national development programmes.
Why in the news?
3. Correct Option: (a)
 Indian-origin satellite industry expert Aarti
O
Holla-Maini will take over the Director of Explanation:
the United Nations Office for Outer
 Statement 2 is incorrect: The PDI
Space Affairs (UNOOSA) in Vienna.
considers nine themes, including poverty-
free and enhanced livelihood, healthy
SC
2. Correct option: (b) village, child-friendly village, water-
sufficient village, clean and green village,
Explanation: self-sufficient infrastructure, socially just
 Statement 1 is incorrect: The scheme has and secured villages, good governance, and
two objectives: women-friendly village.
 The objective of the first component of  It does not consider education as one of
the scheme is to maximise domestic its criteria.
electrolyser manufacturing capacity and
GS

achieve lower production costs.  Statement 3 is incorrect: It is formulated


by the Centre.
 The second component of the incentive
scheme focuses on Green Hydrogen Supplementary notes
Supplementary notes Panchayat Development Index
Strategic Intervention for Green  A ‘Panchayat Development Index’ has
Hydrogen Transition (SIGHT) been formulated by the Centre to evaluate
different indicators responsible for ensuring
 The Ministry of New & Renewable
targeted development in villages.
Energy (MNRE) has announced guidelines
under the National Green Hydrogen  The Index will be a compact statistical tool
Mission for manufacturing electrolysers to measure and monitor development at
and production of green hydrogen. Panchayats level.
 Earlier this year, MNRE launched the  The PDI ranks Panchayats on the basis
National Green Hydrogen Mission, for of scores, and categorises them into four
implementing the Strategic Interventions grades.
for Green Hydrogen Transition (SIGHT).
 Those with score under 40 per cent fall in
 The Strategic Interventions for Green grade D, 40-60 per cent in grade C, 60-75
O

Hydrogen Transition (SIGHT) programme per cent in grade B, 75 to 90 per cent in


C

is a major financial measure under the


S.

category A, while those scoring above 90 per


TE

Mission.
cent will be categorised as A+.
O
FN

2 PPP-PTS-4392/082023/17
D
.P
W
W
W
https://www.pdfnotes.co/

Why in News?  The scheme is to incentivise States and


Union Territories to promote alternative
 Recently, the Union Minister of State
fertilisers and counteracted use of chemical
for Panchayati Raj released the Report
fertilisers.
on Panchayat Development Index (PDI)
at the National Workshop on Panchayat  This scheme strives to decrease the usage of
Development Index in New Delhi. chemical fertilisers in the states.
Why in News?
4. Correct Option: (b)  Cabinet Committee on Economic Affairs
(CCEA), approved the PM-PRANAM scheme,
Explanation: aimed at restoring and nurturing Mother
 Statement 3 is incorrect: Short-lived Earth through the use of biofertilizers.
halogens increase methane’s lifetime
in the atmosphere by destroying hydroxyl
radicals (OH.
6. Correct Option: (b)
 OH is a sink as it is known to break down Explanation:
this greenhouse gas.  Option (b) is correct: The zombie drug
 These short-lived halogens increased causes low blood pressure and slow
global methane burden by 14 per cent heart rate.
and 9 per cent for pre-industrial and Supplementary notes

RE
present-day conditions, respectively.
 Zombie drug xylazine is an animal
Supplementary notes tranquilizer, also known as “tranq”.
Short-lived halogens  Laced with fentanyl and sold on the streets,
it is known for leaving addicts hunched over
 Oceans do more than just absorbing carbon like zombies, besides rotting flesh at the site
dioxide and moderating the climate. They of injection.
also cool the planet by releasing short-lived
O
halogens such as chlorine, bromine and
 Besides rotting the flesh, this zombie drug
causes low blood pressure, slow heart rate
iodine, a new study has found.
and difficulty breathing.
 Currently, these halogens contribute 8-10
 Since xylazine is not an opioid, it also does
percent of cooling. This could increase to 18-
not respond to the overdose-reversing drug
SC
31 per cent by 2100, the study published in naloxone, which is commonly sold under the
Nature projected. brand name Narcan.
 Short-lived halogens, which have a lifetime
Why in News?
of less than six months in the atmosphere,
are naturally produced by the oceans.  USA’s zombie drug epidemic has claimed
nine lives and led to 150 cases of overdose
Why in News? in a Florida county in the last 18 months
 According to a recent study, the oceans alone
GS

contribute 8-10% to the cooling of the planet


by emitting short-lived halogens.
7. Correct Option: (d)
Explanation:
5. Correct Option: (c)
 Option (d) is correct: The Ol Chiki script
Explanation: was created in 1925 as a way to write
 Both statements are correct Santhali.

Supplementary notes Supplementary notes


 The Ol Chiki script was created in 1925 by
Programme for Restoration, Awareness, Raghunath Murmu (1905-1982), writer and
Nourishment and Amelioration of Mother teacher from what is Mayurbhanj State (now
Earth’ (PRANAM) part of Odisha) in India, as a way to write
 Recently, the Cabinet gave its nod to PM Santli a Munda language spoken mainly
PRANAM — the scheme designed for in Jharkhand and West Bengal states in
balanced use of fertilisers by farmers. With northern India, and also in northwestern
this scheme the reliability of chemicals by Bangladesh, eastern Nepal and Bhutan.
the farmers will be reduced.  Raghunath Murmu created Ol Chiki as a
way to promote Santali culture, and also
O

 The Yojana’s full form is ‘Programme for


because he thought that Santali should have
C

Restoration, Awareness, Nourishment and


S.

its own script, like other major languages of


Amelioration of Mother Earth’ (PRANAM).
TE

India.
O
FN

PPP-PTS-4392/082023/17 3
D
.P
W
W
W
https://www.pdfnotes.co/

 The script was first publicized in 1939 at  The societies whose primary objective is
the Mayurbhanj State exhibition. to serve the interests of its members in a
 Murmu published over 150 books in Santali particular State are governed by the co-
in the Ol Chiki script, including novels, operative societies Act of that specific state.
poetry, drama, grammars, dictionaries and  While, a Society whose primary objective is
other information about the language and to serve the interests of its members in more
script. than one state, is governed by the Multi-
State Co-operative Societies Act of 2002.
 A number of other people have also published
books in Ol Chiki. Supplementary notes
 Ol Chiki is also known as Ol Cemet’, Ol Cooperative Societies
Ciki, Ol or the Santali alphabet. Santali is
also written with the Latin, Odia, Bengali  Cooperatives are collective businesses
and Devanagari alphabets. jointly owned by participating members
who share profits and losses. The popular
Why in News? milk brand Amul, for instance, runs on a
 The Prime Minister India for saluting the co-operative model. So does India’s largest
immortal tribal revolutionaries on Hul fertilizer maker, IFFCO. Cooperative banks
Diwas used Ol Chiki script of Santhali are another key segment that serves both
language. the rural and urban economy.

RE
 A co-operative society is often a voluntary
association of individuals who come together
8. Correct Option: (c) with the intention to work together and to
Explanation: promote their economic interest.
 Option (c) is correct: The Solicitor  These societies work on the principle of self-
General of India (SGI) is the second- help as well as mutual help.
highest law officer in the country and serves  The primary goal is to provide support to
O
as the chief legal advisor to the Government the members.
of India.
 Nobody leaves a co-operative society without
Supplementary notes earning a profit. People of the same interest
come forward as a group, pool their resources,
Solicitor General of India (SGI)
SC
utilise these resources in the best possible
 The SGI represents the government in legal manner and derive a common benefit out
matters and provides legal advice and of it. It is an association of persons who
opinions on various issues. voluntarily share their resources for using
them for the mutual welfare of its members
 They assist the Attorney General of India itself.
in representing the government before the
courts, including the Supreme Court, and  A co-operative society is formed for the
play a crucial role in the formulation and promotion of thrift, self-help and mutual
GS

interpretation of government policies and assistance of the members.


laws. Why in News?
 The Solicitor General also represents  The National Co-operative Union of
the government in significant cases and India (NCUI) and the National Co-
provides legal opinions on matters referred operative Development Corporation
to them by government departments and (NCDC) are the essential agencies working
ministries. for the promotion of co-operative movement
Why in News? in India.

 The Appointments Committee of the Cabinet


has approved re-appointment of the Tushar 10. Correct Option: (a)
Mehta as Solicitor General of India.
Explanation:
 Option (a) is correct
9. Correct Option: (b)
Supplementary notes
Explanation:
Elucid mission
 Statement 3 is incorrect: A co-operative
society may be governed by the respective  The European Space Agency (ESA) has a
O

state’s Co-operative Societies Act or by the new mission, Euclid, set to explore the dark
C

universe. Euclid is named after an ancient


S.

Multi-State Co-operative Societies Act,


TE

2002. Greek mathematician.


O
FN

4 PPP-PTS-4392/082023/17
D
.P
W
W
W
https://www.pdfnotes.co/

 It launched successfully from Cape Supplementary notes


Canaveral, Florida.
Santhal Rebellion
 The mission’s goal is to create the largest,
 The Santhal Hul (revolution) was a tribal
most accurate 3D map of our universe. Also,
revolt and holds a lot of significance to
scientists will use it to observe galaxies out
the Santhal tribe – the third-largest
to 10 billion light-years, covering more than
scheduled tribe community in India
1/3 of the sky.
after the Gonds and Bhils.
 These observations should help astronomers
 The Santhal Hul took place between 1855
to better understand the universe’s expansion
and 1856 and at the forefront of it were the
and structure over time, providing insight to
Santhal Adivasis and lower-caste peasants
the nature of dark matter and dark energy.
fighting against the atrocities of the upper-
Why in News? caste landlords, moneylenders, traders,
police, and administrative officials from
 Recently, the ESA’s Euclid space telescope
the East India Company in the erstwhile
lifted off from Cape Canaveral in Florida.
Bengal presidency.
 In the late 1700s, the Santhals were driven
11. Correct Option: (b) out of Birbhum by the wealthy zamindars
(landlords) and were forced to settle in an
Explanation:
area known as Santhal Parganas which is

RE
 Statement 3 is incorrect: Countries in present-day Jharkhand. They cleared the
manage their own maritime territory dense jungles and were provided land for
and exclusive economic zones, while the settlement in the foothills for rent. Once the
high seas and the international ocean land was cleared, their rent was increased
floor are governed by the United Nations by the zamindars and the moneylenders
Convention on the Law of the Seas. took control of their land and forced them
It is considered to apply to states regardless into bonder labour.

O
of whether or not they have signed or  The Santhals believed that since they cleared
ratified it. the land and inhabited it, it belonged to them.
However, it was not easy to raise their voices
Supplementary notes
against the landlords and moneylenders
SC
Deep sea mining and the British administration paid no heed
to their pleas.
 Deep sea mining involves removing mineral
deposits and metals from the ocean’s  On 30 June 1855, the Murmu brothers
seabed. (Sidho Murmu and Kanhu Murmu)
mobilised around 10,000 people against
 There are three types of such mining:
the zamindars, moneylenders, and British
taking deposit-rich polymetallic nodules
in the village of Bhognadih in present-day
off the ocean floor, mining massive seafloor
Jharkhand. It is considered a historic event
GS

sulphide deposits and stripping cobalt


in Santhal history, which kicked off the
crusts from rock.
revolution.
 These nodules, deposits and crusts contain
 The Santhals started marching to Calcutta
materials, such as nickel, rare earths, cobalt
and were joined by other tribes and lower-
and more, that are needed for batteries and
caste groups. A Santhal head man, Harma
other materials used in tapping renewable
Desmanjhi, was arrested in Panchkatia in
energy and also for everyday technology
present-day West Bengal and this led to the
like cell phones and computers.
rebellion spreading further.
Why in News?  The Murmu brothers – Sidho, Kanhu,
 The International Seabed Authority (ISA) is Chand and Bhairab – were among those
preparing to allow Deep Sea Mining in the who led the revolt. But the women also
International Seabed, including mining for played an important role. Phulo Murmu and
minerals needed for Green Energy. Jhalo Murmu, sisters from the same family,
participated in the Hul, inspiring women to
join the rebellion.
12. Correct Option: (b)  In the conflict, the tribes fought British
Explanation: troops with bows and arrows.
O

Statement 2 is incorrect: The Santhals In November 1855, martial law was


C

 
S.

were repressed and the movement came to introduced to curb the revolt and the Hul
TE

an end in 1856. was quashed by early 1856.


O
FN

PPP-PTS-4392/082023/17 5
D
.P
W
W
W
https://www.pdfnotes.co/

 It led to the formation of the Santhal  The Area and its resources are the common
Parganas and the passing of the Santhal heritage of humankind. The Area covers
Pargana Tenancy Act, 1876, which around 54 per cent of the total area of the
outlawed the transfer of Adivasi land to world’s oceans.
non-Adivasis.
Why in News?
Why in News?  The International Seabed Authority’s
 Prime Minister Narendra Modi greeted member nations has agreed on a two-year
the community on Hul Diwas on June roadmap for the adoption of deep sea mining
30, remembering the sacrifice of Adivasis regulations, despite conservationists’ calls
in their fight against British colonial for a moratorium on mineral extraction they
authorities. say would avert marine threats.
 The Santal rebellion or ‘Hul’ – literally,
revolution – began in 1855, two years before 14. Correct Option: (d)
the the uprising of 1857, often referred to as
“the first war for Indian independence”. Explanation:
 All of the statements are correct
13. Correct Option: (a) Supplementary notes
Explanation: Soil microbiomes

RE
 Statement 2 is incorrect: As of 2023,  Soil micro biomes play a crucial role
the Framework Agreement of the ISA was in maintaining healthy water and
signed by multiple countries, such as the environmental stability.
United States, Japan, France, India, Brazil,
Australia, Argentina, Chile and Algeria,  Moreover, they underpin global food
and ratified by 52 other countries. security that eventually affects the overall
sustainability of terrestrial life in multiple
Supplementary notes ways.
O
International Seabed Authority (ISA)  These microbiomes provide a habitat for
microorganisms that benefit the environment
 The International Seabed Authority (ISA) is by delivering important ecosystem and host
an autonomous international organization functions.
established under the 1982 United Nations
SC
Convention on the Law of the Sea (UNCLOS)  But they also work as a reservoir of human
and the 1994 Agreement relating to the pathogens that induce antibiotic resistance
Implementation of Part XI of the United genes (ARGs) and are sources of organic and
Nations Convention on the Law of the Sea inorganic pollutants.
(1994 Agreement).  Soil microbiomes impact human and animal
 ISA is the organization through which States immune systems by interacting with them
Parties to UNCLOS organize and control through food chains.
all mineral-resources-related activities in They also directly influence the quality of
GS


the Area for the benefit of humankind as a the environment through air and water.
whole.
Why in the news?
 In so doing, ISA has the mandate to ensure
the effective protection of the marine  A group of soil experts and researchers have
environment from harmful effects that may called for including soil microbiomes under
arise from deep-seabed-related activities. the One Health goals that function at the
nexus of human, wildlife and the shared
 ISA, which has its headquarters in environment health.
Kingston, Jamaica, came into existence
on 16 November 1994, upon the entry
into force of UNCLOS. It became fully 15. Correct Option: (c)
operational as an autonomous international
organization in June 1996, when it took Explanation:
over the premises and facilities in Kingston,  Option (c) is correct: Bangladesh is the
Jamaica, previously used by the United observer.
Nations Kingston Office for the Law of the
Sea. Supplementary notes
 In accordance with UNCLOS, Article 156(2), Colombo Security Conclave (CSC)
all States Parties to UNCLOS are ipso facto The CSC evolved out of trilateral meetings
O


members of ISA. As of 1 May 2020, ISA has
C

between NSAs and Deputy NSAs from


S.

169 Members, including 168 Member States India, Maldives, and Sri Lanka, starting in
TE

and the European Union. 2011.


O
FN

6 PPP-PTS-4392/082023/17
D
.P
W
W
W
https://www.pdfnotes.co/

 Since its revival and re-branding as the CSC  The absence of quasars closer to Earth does
in 2020, Mauritius was added as a member not mean that there were never quasars
of the grouping, with Bangladesh and the in our region of the Universe, but instead
Seychelles as observers. means that quasars existed when the
 A secretariat for the group was established universe was younger.
in Colombo in 2021.
Why in News?
 The Conclave underlines regional
cooperation and shared security objectives  India’s Giant Metrewave Radio Telescope
concerning all littoral nations in the Indian (GMRT), along with four other international
Ocean Region (IOR). radio telescopes, has succeeded in detecting
fringes from an extremely bright quasar
 It aims to make maritime security, marine
pollution response and maritime search & located 12.3 billion light years away.
rescue priorities for the region.
Why in News? 17. Correct Option: (c)
 The seventh Deputy NSA level meeting of Explanation:
the Colombo Security Conclave (CSC) was
hosted on 11th July by Maldives amid efforts  Both statements are correct
to keep the Indian Ocean Region stable and Supplementary notes
peaceful.

RE
 Member States India, Maldives, Mauritius Bharat 6G Alliance (B6GA)
and Sri Lanka met in the virtual format.  The primary objective of B6GA is to
Bangladesh and Seychelles participated as understand the business and societal needs
Observers. of 6G beyond technology requirements,
foster consensus on these needs, and
16. Correct Option: (b) promote high-impact open research and

Explanation:
O development (R&D) initiatives.
 B6GA aims to bring together Indian start-
 Statement 3 is incorrect: A quasar is ups, companies, and the manufacturing
an extremely active and luminous type of ecosystem to establish consortia that drive
active galactic nucleus (AGN). All quasars
SC
the design, development and deployment of
are AGNs, but not all AGNs are quasars.
6G technologies in India.
Supplementary notes  By accelerating standards-related patent
Quasars creation within the country and actively
contributing to international standardization
 Quasars are a subclass of active galactic
organizations such as 3GPP and ITU, B6GA
nuclei (AGNs), extremely luminous
galactic cores where gas and dust falling seeks to position India at the forefront of 6G
innovation.
GS

into a supermassive black hole emit


electromagnetic radiation across the entire  One of the key goals of B6GA is to facilitate
electromagnetic spectrum. market access for Indian telecom technology
 The gas and dust become luminous as a products and services, enabling the
result of the extreme gravitational and country to emerge as a global leader in 6G
frictional forces exerted on them as they fall technology.
into the black hole.
 To achieve this, these efforts shall promote
 Quasars are amongst the most luminous technology ownership and indigenous
objects in the known Universe, typically manufacturing, create a culture of
emitting thousands of times more light than technology co-innovation, reduce imports,
the entire Milky Way.
boost export opportunities and augment
 They are distinguished from other AGNs creation of Intellectual Property.
by their enormous luminosity, and their
enormous distances from Earth. Why in News?
 As the speed of light is finite, objects  Recently, the Department of
observed from Earth are seen as they were Telecommunications (DoT) under the
when the light we see left them. The nearest Ministry of Communications launched
quasars to Earth are still several hundred the Bharat 6G Alliance (B6GA) to
O

million light-years away, meaning that they foster innovation and leadership in 6G
C
S.

are observed now as they were 600 million technology, the next frontier of wireless
TE

years ago. communication.


O
FN

PPP-PTS-4392/082023/17 7
D
.P
W
W
W
https://www.pdfnotes.co/

18. Correct Option: (a)  Tele-MANAS is a two-tier system;


 Tier 1 comprises State Tele MANAS
Explanation:
cells, which includes trained counsellors
 Option (a) is correct: Data scraping is and mental health specialists.
the automated process of extracting large
 Tier 2 comprises specialists at District
amounts of data from websites or online
Mental Health Programme (DMHP)/
sources.
Medical College resources for physical
Supplementary notes consultation and/or eSanjeevani for
audio visual consultation.
Data scraping
 State Tele MANAS Counsellor- Tier 1
 Data scraping, also known as web scraping, Counsellors (TMC): The first point of contact
is the process of importing information for people reaching out to Tele MANAS.
from a website into a spreadsheet or local They will be handling calls from individuals
file saved on your computer. It’s one of the in shifts and working from the State Tele
most efficient ways to get data from the MANAS Cell.
web, and in some cases to channel that data
to another website. Popular uses of data Why in News?
scraping include:  The union health ministry said the Tele-
 Research for web content/business Manas helpline under the National Tele
intelligence Mental Health Programme has received

RE
over 200,000 calls since its launch in October
 Pricing for travel booker sites/price
2022.
comparison sites
 The number of calls has doubled in just
 Finding sales leads/conducting market
three months.
research by crawling public data sources
(e.g. Yell and Twitter)
 Sending product data from an e-commerce 20. Correct Option: (a)
O
site to another online vendor (e.g. Google
Shopping)
Explanation:
 Statement 2 is incorrect: In India, the
Why in the news? largest population of the chinkara is found
Google updates its privacy policy to allow in the Thar desert. According to a study in
SC

data scraping for AI training. 2001, the Indian chinkara population was
estimated at 100,000 with 80,000 living in
the Thar Desert.
19. Correct Option: (b)
 Statement 3 is incorrect: The chinkara
Explanation: is assessed by the IUCN Red List as least
concern and included in the CITES
 Statement 2 is incorrect: The programme
Appendix III, so trade is allowed only on
includes a network of 23 tele-mental
presentation of the appropriate permits or
GS

health centres of excellence, with


certificates.
NIMHANS being the nodal centre and
International Institute of Information Supplementary notes
Technology-Bangalore (IIITB) providing
technology support. Indian gazelle
 Indian Institute of Technology (IIT)  The chinkara or Indian gazelle (Gazella
Bengaluru and National Health Systems bennettii) is a relatively small gazelle, its
Resource Centre (NHRSC) will provide the weight being about 23 kg and height 65 cm.
technical support.  The species has a reddish-buff summer coat
with smooth, glossy fur. In winter, the white
Supplementary notes
belly and throat fur is in greater contrast.
Tele-MANAS  The range states of the species are India,
 The Government launched India’s first Tele- the Islamic Republic of Iran, Pakistan and
MANAS chat-bot in the Union Territory of possibly Afghanistan.
Jammu & Kashmir, which will ensure round  The chinkara inhabits arid areas, including
the clock services of health counsellors, sand deserts, flat plains and hills, dry scrub
clinical psychologists and consultants. and light forest. They can go for a long time
 The Centre had in Budget 2022 announced without water, so they are adapted to live
O

the National Tele Mental Health Programme in very dry areas. It is mostly a solitary
C
S.

of India, Tele Mental Health Assistance and animal, but sometimes forms small groups
TE

Networking across States (Tele-MANAS). of 2 – 4 individuals.


O
FN

8 PPP-PTS-4392/082023/17
D
.P
W
W
W
https://www.pdfnotes.co/

 Most of the global population is found in the 22. Correct Option: (a)
Thar Desert of western India.
Explanation:
 The current estimates suggest around
100,000 individuals in India, 1,300 in  Option (a) is correct: In an extractive
Iran and significant uncertainties about economy, the focus is predominantly on
the population numbers in Pakistan and extracting and exporting raw materials
or commodities, often at the expense of
Afghanistan.
sustainable development, environmental
 The major threats include illegal hunting preservation, and the well-being of local
for meat and sometimes trophies in communities.
Afghanistan, Iran and Pakistan. Habitat
loss through overgrazing, conversion to Supplementary notes
agriculture and industrial development is Extractive Economy
also a factor.
 An extractive economy is a specific type
Why in News? of economic system that heavily relies on
the extraction and exploitation of natural
 In a first-of-its-kind a sessions court in
resources as a primary driver of economic
Hanumangarh, Rajasthan ordered payment
activity.
of the half the fine amount imposed on a
killing of a Chinkara to the informer as a  In an extractive economy, the focus is
predominantly on extracting and exporting

RE
prize for helping in the detection of crime
against wildlife. raw materials or commodities, often at
the expense of sustainable development,
environmental preservation, and the well-
21. Correct Option: (d) being of local communities.
 Characteristics of an extractive
Explanation:
economy include:
 Option (d) is correct: All particles have
O
the potential to cool down the earth when
 Natural Resource Dependency
pumped into the upper atmosphere.  Limited Economic Diversification
 Foreign Ownership and Control
Supplementary notes
 Environmental Degradation
SC
Stratospheric aerosol injection
 Social Inequality
 Sulfur dioxide, aluminum, calcium
carbonate, and, most poetically, diamond Why in the news?
dust ‘Sun blocking’ technologies - also  In a veiled attack on China, External Affairs
known as ‘solar radiation modification’ aims Minister S Jaishankar has said that India
to cool down the earth by reflecting sunlight is not an “extractive economy” and does not
back into space. pursue narrow economic activities solely
for its benefit in the resource-rich African
GS

 One idea involves pumping sun-blocking


continent.
particles into the upper atmosphere. This
process of ‘stratospheric aerosol injection’
would involve planes spraying an aerosol 23. Correct Option: (d)
like sulfur dioxide into the stratosphere.
Explanation:
 The sulfur dioxide moves into the
stratosphere and combines with water to  Option (d) is correct: The ToLCV is a
form sulfuric acid aerosols. The sulfuric plant virus that infects tomatoes. It is
acid makes a haze of tiny droplets in the transmitted by whitefly Bemisia tabaci. It
can affect plants at any stage of growth but
stratosphere that reflects incoming solar
they are mostly vulnerable when they are
radiation, causing cooling of the Earth’s
young.
surface.
 Aerosols are small particles or droplets that Supplementary notes
float in the air. They are emitted by both Tomato leaf curl virus (ToLCV)
natural events and human activities. Some
 Tomato leaf curl virus (ToLCV) is a plant
aerosols cool the climate.
virus that affects tomato plants and other
Why in the news? crops in the Solanaceae family, such as
peppers, eggplants, and potatoes.
O

 A White House report confirms that the


C

It is a member of the Begomovirus genus


S.

US is open to researching solar radiation 


TE

modification, or sun blocking. and is primarily transmitted by whiteflies.


O
FN

PPP-PTS-4392/082023/17 9
D
.P
W
W
W
https://www.pdfnotes.co/

 Symptoms of Tomato leaf curl virus 25. Correct Option: (b)


infection include leaf curling, stunted
growth, yellowing or bronzing of leaves, Explanation:
reduced fruit production, and overall decline  Statement 3 is incorrect: Using cluster
in plant health. The virus interferes with bombs is not a war crime.
the plant’s ability to carry out essential
functions, ultimately impacting crop yield Supplementary notes
and quality. Cluster bombs
 ToLCV is a significant concern for tomato  Cluster munitions, which are also known
farmers as it can cause substantial economic as cluster bombs, are canisters that contain
losses. tens to hundreds of smaller bomblets, also
 It is prevalent in many regions of the world, called submunitions.
particularly in tropical and subtropical  The canisters can be launched using
areas where whiteflies are abundant. missiles, dropped from aircraft, or fired from
 The virus can spread rapidly within and naval guns, artillery, or rocket launchers.
between fields, leading to widespread  At a prescribed height, the canisters break
infections if not properly managed. open depending upon the intended target,
and the bomblets inside it spread out over
Why in News? that area.

RE
 Farmers in Karnataka’s Kolar are blaming  The bomblets fuse with a timer and explode
the tomato leaf curl virus (ToLCV) for crop closer to or on the ground and spread
losses due to which vegetable prices have shrapnel that is designed to take out
skyrocketed. armoured vehicles such as tanks and kill
 While in Maharashtra, tomato growers say troops.
cucumber mosaic virus (CMV) and tomato
Why in News?
mosaic virus (ToMV) are affecting their
produce United States (US) has made the decision to
O

provide cluster munitions to Ukraine as
part of a new military aid package worth up
24. Correct Option: (c) to USD 800 million.
Explanation:
SC
 Both statements are correct 26. Correct Option: (b)
Supplementary notes Explanation:
 Statement 2 is incorrect: Disturbance
African Union (AU) means an area of “disturbed” or reduced air
 The African Union (AU) is a pressure.
regional intergovernmental
organisation that replaced the Supplementary notes
GS

Organisation of African Unity (OAU). Western disturbance (WD)


The African Union (AU) is a continental
 A western disturbance (WD) is an extra-
organization consisting of 55 member states
tropical storm which originates in the
in Africa.
Mediterranean region, the meaning of WD
 It was established on May 26, 2001, in Addis lies in its name.
Ababa, Ethiopia, with the goal of promoting
 The disturbance travels from the “western”
unity, solidarity, and cooperation among
to the eastern direction. Disturbance
African nations. means an area of “disturbed” or reduced air
 The organisation’s main goal is to promote pressure. Equilibrium exists in nature due
and accelerate the political and socio- to which the air in a region tries to normalise
economic integration of the continent. its pressure.
 It also focuses on encouraging international  In the term “extra-tropical storm”, storm
cooperation, protecting human rights and refers to low pressure. “Extra-tropical”
promoting and defending common African means outside the tropics. As the WD
positions on key issues. originates outside the tropical region, the
word “extra-tropical” has been associated
Why in the news? with them.
 Prime Minister Narendra Modi has been  A WD is associated with rainfall, snowfall
O

pushing for the African Union – a continental and fog in northern India. Upon its arrival
C
S.

body comprising 55 member states – to join in Pakistan and northern India, clouds
TE

the G20. along with rain and snow also arrive.


O
FN

10 PPP-PTS-4392/082023/17
D
.P
W
W
W
https://www.pdfnotes.co/

 The moisture which WDs carry with them  This art form was first performed some
comes from the Mediterranean Sea and/or 150 years back by the people of Vedar and
from the Atlantic Ocean. Pulayar communities. Seethakali is based
on certain episodes taken from the Indian
Why in News? epic Ramayana.
 The weather department has attributed  Mythic characters such as Rama, Seetha,
the heavy rainfall over Chandigarh and its Ravana and Hanuman come alive in
adjoining areas to a Western Disturbance Seethakali performances that portray the
(WD), which hit the northern parts of India tale of Seetha’s journey, from the time she
on July 8. accompanied Rama to the woods till her
 According to AK Singh, officiating director ascent to the heavens.
of the Meteorological Centre, Chandigarh,
and the Western Disturbance will withdraw Why in news?
from the region.  The Perinad Seethakali Sangham currently
has artistes from all walks of life and the 20-
member group is all set to perform outside
27. Correct Option: (b) Kerala for the first time.
Explanation:
 Statement 3 is incorrect: It has 32- 29. Correct Option: (c)
members and the Champions Group
Explanation:

RE
comprising representatives of Bangladesh,
Barbados, Denmark, Germany,  Statement 4 is incorrect: The disease is
Indonesia, and Senegal. (They are not more common in adults and males but can
members) affect people of all age groups, as per the
 Statement 4 is incorrect: India is also the World Health Organization (WHO).
member of Global Crisis Response Group Supplementary notes
(GCRG)
Guillain-Barré Syndrome
Supplementary notes
O  Guillain-Barré Syndrome is a disorder
Global Crisis Response Group (GCRG) where a person’s immune system attacks
 The Global Crisis Response Group (GCRG) their body’s peripheral nerves.
was set up by the UNSG in March 2022  The immune system is the body’s natural
SC
to address urgent and critical global defence against illness and infection. The
issues pertaining to interlinked crises in peripheral nervous system consists of the
food security, energy, and finance and to network of nerves outside the brain and
coordinate a global response. spinal cord.
 The GCRG is overseen by the Champions  In GBS, the myelin sheath – the layer of
Group comprising of HOS/ HOGs of fat and protein encasing the nerve cells –
Bangladesh, Barbados, Denmark, Germany, becomes inflamed and can hardly carry
Indonesia and Senegal. stimuli.
GS

Why in News?  GBS usually occurs after an infection.


According to the United States’ Centers
 India has joined the champions group of the for Disease Control and Prevention (CDC),
Global Crisis Response Group (GCRG). about two in every three people who were
diagnosed with the neurological disease
28. Correct Option: (a) had diarrhea or a respiratory illness several
weeks before the onset of GBS symptoms.
Explanation:  The symptoms of GBS mostly start within
 Option: (a) is correct: Seethakali is a three weeks of an infection.
traditional folk art form that originated in  People infected with cytomegalovirus,
the Western Ghats of Karnataka, India. This Campylobacter jejuni bacteria, Epstein-
art form is often associated with the worship Barr virus, Zika virus, or other viruses
of the goddess Seetha and is characterized could also be affected by GBS.
by vibrant costumes, intricate masks, and
rhythmic movements.  As per the WHO, Zika virus infection is a
“trigger” of Guillain-Barré syndrome.
Supplementary notes
Why in News?
Seethakali
 Peru has declared a national health
The unique and flamboyant Seethakali is a
O


emergency for 90 days due to a surge in the
centuries-old folk art form that is believed
C

cases of Guillain-Barré Syndrome (GBS), a


S.

to have originated at Perinad in Kollam


TE

district. rare neurological disorder.


O
FN

PPP-PTS-4392/082023/17 11
D
.P
W
W
W
https://www.pdfnotes.co/

30. Correct Option: (c) 31. Correct Option: (d)


Explanation: Explanation:
 All statements are correct  Option (d) is correct: Phosphine is a
compound made from phosphorus and
Supplementary notes
hydrogen, and on Earth it’s only natural
National Securities Depository Ltd. source is tiny microbes that live in oxygen-
(NSDL) free environments.
 Central Depositories Services India Ltd. Supplementary notes
(CDSL) and National Securities Depository
Ltd. (NSDL) are both government registered Phosphine
share depositories in India.  Phosphine on Earth is developed by bacteria
 NSDL, one of the largest Depositories in that live in very low-oxygen environments.
the World, established in August 1996 has In contrast, Phosphine has been found deep
established a state-of-the-art infrastructure into the clouds of Venus.
that handles most of the securities held and  Phosphine is a colorless, flammable, and
settled in dematerialized form in the Indian
explosive gas at ambient temperature that
capital market.
has the odor of garlic or decaying fish.
 Although India had a vibrant capital
 Small amounts occur naturally from the
market which is more than a century old,

RE
break-down of organic matter. It is slightly
the paper-based settlement of trades caused
substantial problems like bad delivery and soluble in water.
delayed transfer of title, etc.  Phosphine is used in semiconductor and
 The enactment of Depositories Act in August plastics industries, in the production of a
1996 paved the way for establishment of flame retardant, and as a pesticide in stored
NSDL. grain.
 Using innovative and flexible technology Why in News?
O
systems, NSDL works to support the
 Three years ago, astronomers announced
investors and brokers in the capital market
that they had detected phosphine, a molecule
of the country.
associated with biological activity on Earth,
 NSDL aims at ensuring the safety and in the clouds of Venus. This unexpected
soundness of Indian marketplaces by
SC
discovery ignited a wave of excitement and
developing settlement solutions that debate within the scientific community.
increase efficiency, minimize risk and
reduce costs.  Now, the team behind the find has unveiled
new evidence, revealing that phosphine
 At NSDL, we play a central role in developing has been detected even deeper in Venus’s
products and services that will continue to atmosphere, all the way down to the
nurture the growing needs of the financial clouds.
services industry. In the depository system,
securities are held in depository accounts,
GS

which is more or less similar to holding 32. Correct Option: (c)


funds in bank accounts.
 Transfer of ownership of securities is done
Explanation:
through simple account transfers.  Both statements are correct
 This method does away with all the risks Supplementary notes
and hassles normally associated with
paperwork. Consequently, the cost of Small Satellite Launch Vehicle (SSLV)
transacting in a depository environment  SSLV is the new small satellite launch
is considerably lower as compared to vehicle developed by ISRO to cater the
transacting in certificates. launch of small satellites up to 500 kg to
 NSDL provides bouquet of services to Low Earth Orbits on ‘launch-on-demand’
investors, stock brokers, custodians, issuer basis. It is configured with three solid stages
companies etc. through its nationwide 87 t, 7.7 t and 4.5 t respectively.
network of Depository Partners.
 SSLV is a 34 m tall, 2 m diameter vehicle
Why in News? having a lift-off mass of 120 t.
 National Securities Depository Limited  A liquid propulsion-based Velocity
(NSDL), the largest depository in India, Trimming Module (VTM) achieves desired
is set to launch its public issue and has velocity for the insertion of the satellites
O

filed preliminary IPO papers for it with into the intended orbit. SSLV is capable of
C
S.

the capital markets regulator Securities & launching Mini, Micro, or Nanosatellites
TE

Exchange Board of India (SEBI). (10 to 500 kg mass) to a 500 km orbit.


O
FN

12 PPP-PTS-4392/082023/17
D
.P
W
W
W
https://www.pdfnotes.co/

 It provides low-cost access to Space, offers 34. Correct Option: (c)


low turn-around time, facilitates flexibility
in accommodating multiple satellites and Explanation:
demands minimal launch infrastructure.  Statement 3 is incorrect: The wall cloud
is much smaller and more compact than a
 The SSLV is the sixth launch vehicle
shelf cloud and is usually under a rain free
developed by the ISRO, following the
cloud base.
Satellite Launch Vehicle-3 (SLV-3),
Advanced Satellite Launch Vehicle (ASLV), Supplementary notes
Polar Satellite Launch Vehicle (PSLV),  According to the US government’s
Geosynchronous Satellite Launch Vehicle National Weather Service (NWS) of the
(GSLV), and Launch Vehicle Mark-3 US government, shelf clouds - also known
(LVM-3). as Arcus clouds - are often associated with
powerful storm systems, and many times
 The SLV-3 and the ASLV have already been
they are reported as wall clouds, funnel
retired.
clouds, or rotation.
Why in the news?  These clouds are sometimes seen beneath
 The Indian Space Research Organisation cumulonimbus clouds, the dense, towering
(ISRO) announced that it will be transferring vertical cloud that causes intense rain.
its Small Satellite Launch Vehicle (SSLV) A shelf cloud will usually be associated with

RE

to the private sector. a solid line of storms.
 The wind will come first with rain following
behind it. It may appear to rotate on a
33. Correct Option: (b) horizontal axis.
Explanation:  Formation: According to the UK Met Office,
 Statement 1 is incorrect: Brucellosis is a when a cold downdraft from a cumulonimbus
cloud reaches the ground, the cold air may
O
bacterial disease caused by various Brucella
spread rapidly along the ground, pushing
species, which mainly infect cattle, swine,
existing warm moist air upwards. As this
goats, sheep and dogs air rises, water vapour condenses into the
Supplementary notes patterns associated with shelf clouds.
SC
Brucellosis Why in News?
 Massive shelf cloud formation captivates
 Brucellosis is a bacterial disease caused
Haridwar amid heavy rains in North India.
by various Brucella species, which mainly
infect cattle, swine, goats, sheep and dogs.
 Humans generally acquire the disease 35. Correct Option: (b)
through direct contact with infected animals, Explanation:
GS

by eating or drinking contaminated animal


 Statement 2 is incorrect: DGNSS is
products or by inhaling airborne agents.
an enhancement system that corrects
 Most cases are caused by ingesting the errors and inaccuracies in the Global
unpasteurized milk or cheese from infected Navigation Satellite System (GNSS). The
goats or sheep. error correction accuracy has been improved
from 5 to 10 meters to less than 5 meters
 Brucellosis is one of the most widespread for 100 Nautical Miles (NM) from Indian
zoonoses transmitted by animals and in coastlines.
endemic areas, human brucellosis has
serious public health consequences. Supplementary notes
 Expansion of animal industries and Differential Global Navigation Satellite
urbanization, and the lack of hygienic System (DGNSS)
measures in animal husbandry and in food  DGNSS is a terrestrial based enhancement
handling, partly account for brucellosis system which corrects the errors and
remaining a public health hazard. inaccuracies in the Global Navigation
Satellite System (GNSS) allowing for more
Why in News? accurate positioning information
 A young girl from Kerala suffered from a  After recapitalization with multiple
O

rare disease called Brucellosis, which is satellite constellations like GPS and Global
C
S.

caused by bacteria that can infect animals Navigation Satellite System (GLONASS),
TE

and humans. DGNSS further increases the availability


O
FN

PPP-PTS-4392/082023/17 13
D
.P
W
W
W
https://www.pdfnotes.co/

and redundancy as per International Supplementary notes


standards and helps the mariners to improve
 To promote greater transparency and
their positioning within 5 meters.
accountability in Panchayat audits, the
 The latest DGNSS system is now able Ministry of Panchayati Raj launched the
to transmit corrections of GPS and
AuditOnline application on 15th April
GLONASS.
2020, enabling online audits of Panchayat
 The DGNSS significantly improves the accounts and further strengthening financial
accuracy of GPS positioning, reducing
errors caused by the atmospheric inferences, management and transparency.
satellite clock drift and other factors.  States have achieved significant progress,
 This is achieved with the help of modern generating over 200,000 Audit Reports in
state of art technology receivers and latest the two last two audit periods 2020–21 and
software. 2021–22.
 The error correction accuracy has been  It is a feat worth commendable on the part
improved from 5 to 10 meters to less than 5
of States and Panchayats.
meters for 100 Nautical Miles from Indian
coastlines.  As on date, 256,795 Panchayats have been
registered; 2,103,058 observations have been
Why in the news?
documented and 211,278 (approximately
 To further the digital initiative in the 80%) PRI Audit Reports have been generated

RE
maritime sector the Union Minister of Ports,
Shipping and Waterways, Shri Sarbananda for audit period 2021–22.
Sonowal inaugurated the indigenous Why in the news?
Differential Global Navigation Satellite
System (DGNSS) ‘SAGAR SAMPARK’.  The Action Taken Report (ATR) Module of
AuditOnline was launched by the Ministry
of Panchayati Raj (MoPR).
36. Correct Option: (c)
O
Explanation:
38. Correct Option: (a)
 All statements are correct
Supplementary notes Explanation:
Statement 2 is incorrect: NATO’s member
SC

Association of World Election Bodies
France is not part of the initiative and has
(A-WEB)
actually spoken against it.
 A-WEB was founded in October, 2013 in
Seoul, Republic of Korea on the shared vision Supplementary notes
among its members of achieving sustainable
democracy around the world through the European Sky Shield Initiative (ESSI)
strengthening of the processes of election
 The initiative aims to create a European
management in member countries.
air and missile defence system through
GS

 A-WEB organises capacity building the common acquisition of air defence


programmes for its member EMBs and
equipment and missiles by European
undertakes Election Visitor and Observation
Programmes in various countries to study nations to strengthen NATO’s Integrated
election management practices and share Air and Missile Defence (IAMD).
knowledge with other member EMBs.  A total of 17 countries have already joined
Why in News? ESSI, first launched in October 2022.
 Chief Election Commissioner Shri Rajiv Why in News?
Kumar led a delegation from the Election
Commission of India (ECI) to attend the  In a sign of changing dynamics within
11th meeting of the Executive Board of Europe, Austria and Switzerland — known
the Association of World Election Bodies for their strong traditions of foreign policy
(A-WEB) in Cartagena, Colombia. neutrality — have joined a regional grouping
that seeks to develop a common air defence
37. Correct Option: (d) shield in the wake of Russia’s attack on
Ukraine.
Explanation:
 Option (d) is correct: The Ministry of
Panchayati Raj launched the AuditOnline 39. Correct Option: (c)
O

application to promote greater transparency


C

Explanation:
S.

and accountability in Panchayat audits in


TE

2020.  Both statements are correct


O
FN

14 PPP-PTS-4392/082023/17
D
.P
W
W
W
https://www.pdfnotes.co/

Supplementary notes  IOM is part of the United Nations system,


as a related organization.
Atal Vayo Abhyuday Yojana (AVYAY)
 Atal Vayo Abhyuday Yojana (AVYAY)  IOM supports migrants across the world,
is a comprehensive initiative aimed at developing effective responses to the
empowering senior citizens in India. shifting dynamics of migration and, as such,
is a key source of advice on migration policy
 The Atal Vayo Abhyuday Yojana (AVYAY),
introduced by the Ministry of Social Justice and practice.
and Empowerment, is a comprehensive  The organization works in emergency
initiative aimed at empowering senior situations, developing the resilience of all
citizens in India. people on the move, and particularly those
 This scheme recognizes the invaluable in situations of vulnerability, as well as
contribution made by the elderly to society building capacity within governments to
and seeks to ensure their well-being and manage all forms and impacts of mobility.
social inclusion.
 The Organization is guided by the principles
 By recognizing the invaluable contributions
of the elderly to society, the government enshrined in the Charter of the United
aims to empower and uplift them, ensuring Nations, including upholding human rights
their active participation and inclusion in for all.
all aspects of life.  Respect for the rights, dignity and well-

RE
 The Department of Social Justice and being of migrants remains paramount.
Empowerment, being the Nodal Department
for welfare of senior citizens, is implementing Why in News?
various programs and schemes.  The International Organization for
 National Action Plan for Senior Citizens Migration (IOM) and the International
(NAPSrc) had been revamped, renamed as Labour Organization (ILO) convened a
Atal VayoAbhyuday Yojana (AVYAY) and workshop with key government stakeholders
subsumed in April 2021.
O
Under the umbrella Scheme, Atal Vayo
in Ukraine, including the Ministry of
 Economy, to build awareness and consensus
Abhyuday Yojana (AVYAY), an Integrated on current and future labour market
Programme for Senior Citizens (IPSrC)
challenges in Ukraine, and the policies and
provides financial assistance to eligible
SC
organization’s for running and maintenance capacities required to address them.
of Senior Citizen Homes/ Continuous Care
Homes to improve the quality of life of the
senior citizens, especially indigent senior
41. Correct Option: (b)
citizens by providing basic amenities, Explanation:
entertainment opportunities and by
encouraging productive and active ageing.  Statement 2 is incorrect: The
Chandrayaan-3 carries an instrument
Why in News? named the Spectro-polarimetry of
GS

 The Atal Vayo Abhyuday Yojana, launched HAbitable Planet Earth (SHAPE) – which
by the Ministry of Social Justice and its predecessor did not.
Empowerment, stands as a testament to
the government’s commitment to the well- Supplementary notes
being and empowerment of senior citizens
in India. ISRO’s Chandrayaan-3 mission
 Chandrayaan-3 is a follow-on mission to
40. Correct Option: (c) Chandrayaan-2 to demonstrate end-to-end
capability in safe landing and roving on the
Explanation: lunar surface. The mission aims to explore
 All of the statements are correct the Moon’s surface, particularly areas that
have been deprived of sunlight for billions
Supplementary notes
of years.
International Organization for Migration
 The three main objectives of Chandrayaan-3
(IOM)
are to land safely on the surface, to
 Established in 1951, the International demonstrate rover operations and to
Organization for Migration (IOM) is the perform scientific experiments on site.
leading inter-governmental organization
O

in the field of migration and is committed  The Chandrayaan-3 mission is split between
C
S.

to the principle that humane and orderly the lander, the rover and the propulsion
TE

migration benefits migrants and society. module payload.


O
FN

PPP-PTS-4392/082023/17 15
D
.P
W
W
W
https://www.pdfnotes.co/

 The lander includes: Supplementary notes


 Chandra’s Surface Thermophysical Right to silence
Experiment (ChaSTE) to measure
thermal conductivity and temperature  The right to not speak against him or to
remain silent is given to an accused; this
on the surface;
right is recognized around the globe.
 Instrument for Lunar Seismic Activity
 In India, it forms a part of Right to self-
(ILSA) to detect moonquakes;
incrimination under Article 20(3) of the
 Langmuir Probe to estimate the density Indian Constitution. It is considered the
and variation of plasma, or superheated duty of the prosecution to prove a person
gas, in the moon’s environment; guilty until proven otherwise he remains an
innocent man.
 A Laser Retroreflector Array (from
NASA) to measure distances using laser  In India, criminal law has vested the citizens
ranging. with this right under various provisions.
 The rover “is a rectangular chassis  Article 20(3) of the Indian Constitution
mounted on a six-wheel rocker-bogie states that no one can be compelled to be a
wheel drive assembly”. The rover sends witness against himself, the right to remain
its communications to Earth through the silent emanates from this very Article.
lander. Rover instruments include:  The provision enables the citizens to enjoy
Alpha Particle X-ray Spectrometer the right against self-incrimination which is

RE

(APXS) to look for elements in the lunar a fundamental canon of law.
soil and rocks; Why in News?
 Laser Induced Breakdown Spectroscope  All accused have a right to silence and
(LIBS) to examine the chemical and investigators cannot force them to speak
elemental composition of the lunar up or admit guilt, the Supreme Court said
surface. recently, emphasizing that the Constitution
accords every person a right against self-
O
 The propulsion module “is a box-like
structure with one large solar panel incrimination.
mounted on one side and a large cylinder
on top that acts as a mounting structure for 43. Correct Option: (c)
the lander,”.
SC
 The propulsion module is more than 2.2
Explanation:
tons (2 tonnes in mass.)  Both statements are correct
 The module’s single experiment is the Supplementary notes
Spectro-polarimetry of Habitable Planet
Earth (SHAPE) investigation that will Gulf Stream
assist with exoplanet searches.  The Gulf Stream is a warm and swift
 The experiment will “gather data on the Atlantic Ocean current.
GS

polarization of light reflected by Earth  The Gulf Stream begins in Florida, before
so that researchers can look for other moving along the eastern coastlines of
planets with similar signatures. the US and Canada and then crossing the
Atlantic Ocean to Europe.
Why in News?
 The Gulf Stream makes the climate of
 India’s third moon mission, Chandrayaan-3, Western Europe warmer far warmer than it
was successfully launched onboard a Launch otherwise would be.
Vehicle Mark-3 (LVM-3) rocket from the
 The Gulf Stream is a small part of something
second launch pad at the Satish Dhawan
called the ‘thermohaline circulation’
Space Centre in Sriharikota at 2.35 pm on or ‘Atlantic Meridional Overturning
July 14. Circulation’.
 Scientists describe the AMOC as a giant
42. Correct Option: (c) ocean conveyor belt that moves water from
Greenland south to beyond the tip of Africa
Explanation: and into the Indian Ocean.
 Option (c) is correct: The right to silence  Warm, salty water near the surface moves
emanates from Article 20(3) of the Indian north and mixes with cold, fresher water
Constitution, which states that no one can near Greenland. As that water cools and
be compelled to be a witness against himself. sinks it drives a slow circulation of the
O

The provision gives an accused the right oceans that is critical to global climate,
C
S.

against self-incrimination -- a fundamental affecting the location of droughts and


TE

canon of law. frequency of hurricanes.


O
FN

16 PPP-PTS-4392/082023/17
D
.P
W
W
W
https://www.pdfnotes.co/

 It also stores heat-trapping carbon dioxide starting from inputs, seeds, certification,
deep in the ocean. The faster it moves, the to the creation of facilities for collection,
more warm water gets sent into the depths aggregation, processing marketing and
to cool. brand building initiative.
 The AMOC itself helps control, among other Why in News?
things, sea level rise and the temperatures
in Europe.  On July 2023 progress made under the
scheme was reviewed with focus on how to
Why in News? liquidate the committed liability of Phase III
 Scientists warn of Gulf Stream collapse. and roadmap for implementation of Phase
IV of the scheme staring 2023-24.
44. Correct Option: (c)  States were requested to suggest the
activities and plans which could be taken up
Explanation: associated with availability of funds under
 Both statements are correct MOVCDNER Scheme.

Supplementary notes
Agriculture Infra Fund (AIF)
46. Correct Option: (c)
 Agriculture Infra Fund (AIF) is a financing Explanation:

RE
facility launched on 8th July 2020 for  Option (c) is correct: Silvopasture is a
creation of post-harvest management specific type of agroforestry practice that
infrastructure and community farm assets.
involves the intentional integration of trees
 Under this scheme, Rs.1 lakh crore is to with pastureland and grazing livestock
be disbursed by the financial year 2025- on the same land area. It is designed to
26 and the interest subvention and credit enhance environmental sustainability,
guarantee assistance will be given till the productivity, and provide various benefits
year 2032-33.
O such as improved animal welfare, carbon
Why in the news? sequestration, and soil conservation.
 The Agriculture Ministry asked banks Supplementary notes
to promote Rs. 1 lakh crore Agriculture
SC
Infrastructure Fund (AIF) for creation of Silvopasture
necessary infrastructure in the farm sector.  Silvopasture is a form of agroforestry.
 It is the deliberate integration of trees and
45. Correct Option: (b) grazing livestock operations on the same
land.
Explanation:
 This regenerative farming method benefits
 Statement 3 is incorrect: The focus of
both the land and animals—think free
GS

the Government in NE and exhorted the


8 states to ensure the complete utilization organic matter (i.e., manure) for the soil and
of funds earmarked for NE states, focus a cool, shady environment for livestock—
on export of organic products to ensure not to mention the farmer, who may use the
maximum benefit to the farmers of the NE trees to supplement income from livestock
region. and vice versa.
Supplementary notes  Silvopasture is the deliberate integration of
trees and grazing livestock operations on the
 Realizing the potential of organic farming
same land. These systems are intensively
in the North Eastern Region of the country
Ministry of Agriculture and Farmer Welfare managed for both forest products and
has launched a Central Sector Scheme forage, providing both short- and long-term
entitled “Mission Organic Value Chain income sources.
Development for North Eastern Region” for  Silvopasture systems are created by
implementation in the states of Arunachal introducing forage into a woodland or
Pradesh, Assam, Manipur, Meghalaya, tree plantation or by introducing trees
Mizoram, Nagaland, Sikkim and into a pasture. Rotational grazing is a
Tripura, during the 12th plan period.
key management activity when using
 The scheme aims at development of certified silvopasture in order to minimize damage to
O

organic production in a value chain mode to trees. Special considerations and planning
C

link growers with consumers and to support


S.

must also be taken into account for long-


TE

the development of entire value chain term tree regeneration.


O
FN

PPP-PTS-4392/082023/17 17
D
.P
W
W
W
https://www.pdfnotes.co/

Why in News?  In addition, the platform also support


 In light of the global deterioration of natural scalable pilots and inform policies aimed at
resources and forests, silvopasture systems accelerating freight electrification in India.
offer a relevant solution to deforestation Why in News?
trends.
 Prominent manufacturing companies and
logistics service providers in India have
47. Correct Option: (a) come together to express a demand for 7,750
Explanation: electric freight vehicles by 2030.

 Option (a) is correct: Lyme disease


is a vector-borne disease caused by the 49. Correct Option: (b)
bite of infected black-legged ticks. It is
characterized by symptoms such as fever, Explanation:
fatigue, headache, muscle and joint aches,  Statement 3 is incorrect: The Export
and a distinctive “bull’s-eye” rash known Preparedness Index (EPI) is released by
as erythema migrans. Lyme disease is NITI Aayog.
prevalent in certain regions of the world,
including parts of North America and Supplementary notes
Europe, and early diagnosis and treatment
are essential to prevent complications. Export Preparedness Index (EPI)

RE
 Export Preparedness Index (EPI) is a
Supplementary notes
comprehensive tool which measures the
Lyme disease export preparedness of the States and UTs
 Vectors are mosquitoes, ticks, and fleas that in India.
spread pathogens.  Exports are vital for simulating economic
 A person who gets bitten by a vector and growth and development in a country, which
O
gets sick has a vector-borne disease. necessitates understanding the factors
which influence export performance.
 Some vector-borne diseases, like plague,
have been around for thousands of years.  The index undertakes a comprehensive
 Others, like Heartland virus disease and analysis of States and UTs across export-
related parameters in order to identify their
SC
Bourbon virus disease, have been discovered
recently. strengths and weaknesses.

Why in News?  Developing the methodology for the index


is an evolving process which constantly
 According to a new study, more than 14 incorporates stakeholder feedback.
percent of the world’s population has had
Lyme disease, the most common tick-borne  EPI assesses the performance of the States
illness. and UTs across four pillars – Policy, Business
Ecosystem, Export Ecosystem, and Export
GS

Performance. Each pillar is composed of


48. Correct Option: (c) sub-pillars, which in turn capture a state’s
Explanation: performance using relevant indicators.

 Both statements are correct  Policy Pillar evaluates states and UTs’
performance based on its adoption of
Supplementary notes the export-related policy ecosystem at
e-FAST a state and district level as well as the
institutional framework surrounding
 The Electric Freight Accelerator for
the ecosystem.
Sustainable Transport – India, or e-FAST
India is India’s first national electric freight  Business Ecosystem assesses the
platform. prevailing business environment in
 The objective of the e-FAST platform is to a state/UT, along with the extent of
bring together different stakeholders from business-supportive infrastructure, and
across the freight ecosystem, to strengthen a state/UTs’ transport connectivity.
partnerships and identify and support  Export Ecosystem focuses on the
innovative freight solutions. export-related infrastructure in a state/
 e-FAST India aims to raise awareness on UT along with the trade support provided
O

freight electrification, supported by on- to the exporters, and the prevalence of


C
S.

ground demonstration pilots and evidence- Research and Development in the state/
TE

based research. UT to foster innovation.


O
FN

18 PPP-PTS-4392/082023/17
D
.P
W
W
W
https://www.pdfnotes.co/

 Export Performance is an output-  It is formed following the Space sector


based indicator which gauges the growth reforms to enable and facilitate the
of a state’s export over the previous year participation of private players.
and analyses its export concentration
and footprint on the global markets.  IN-SPACe is responsible to promote,
enable, authorize and supervise various
Why in News? space activities of non-governmental
 NITI Aayog released the third edition of the entities including building launch vehicles
report titled ‘Export Preparedness Index & satellites and providing space-based
(EPI) 2022’ for States/UTs of India. services; sharing space infrastructure and
premises under the control of DOS/ISRO;
and establishing of new space infrastructure
50. Correct option: (a)
and facilities.
Explanation  The agency acts as an interface between
 Option (a) is correct: A Goldilocks ISRO and Non-Governmental Entities
economy describes an ideal state for an (NGEs) and assesses how to utilize India’s
economy whereby the economy is not space resources better and increase space-
expanding or contracting by too much. A based activities.
Goldilocks economy has steady economic
growth, preventing a recession, but not  It also assesses the needs and demands of
so much growth that inflation rises by too private players, including educational and

RE
much. research institutions, and explores ways
Supplementary notes to accommodate these requirements in
consultation with ISRO.
Goldilocks scenario
Why in News?
 A Goldilocks scenario in an economy refers
to an ideal situation where there is a steady  Indian National Space Promotion and
growth. The economic growth is neither too Authorization Center (IN-SPACe) IN-
high to trigger inflation and nor too low for
O
a slowdown. In simple terms, in a goldilocks
SPACe is an independent nodal agency
under Department of Space for allowing
scenario, the economy is not expanding by space activities and usage of DOS owned
a huge margin with inflation or shrinking facilities by NGPEs as well as to prioritise
into recession.
the launch manifest.
SC
 In this state of the economy, there are
certain features attached. For one, the
unemployment rate in the economy is really 52. Correct Option: (b)
low.
Explanation:
 There is steady growth in the Gross Domestic
Product (GDP) numbers and companies  Option (b) is correct: The G20 EMPOWER
report better earnings. initiative aims to accelerate women’s
 The retail inflation and the interest rates leadership and empowerment in the private
sector through collaboration between G20
GS

are relatively low. The Goldilock scenario


is good for investors as companies perform business leaders and governments. It is in
well and stocks rally. line with India’s women-led development
agenda.
Why in news?
 A steady-growing economy with moderate Supplementary notes
inflation and low unemployment rate can be
G20 EMPOWER initiative
optimal for long-term growth and represents
a Goldilocks economy.  In line with the Centre’s flagship Digital
India programme, the Union minister for
women and child development, Smriti Irani
51. Correct Option: (c) launched G20 EMPOWER Tech Equity
Explanation: Digital Inclusion Platform.
 Both statements are correct  The G20 Alliance for the Empowerment
and Progression of Women’s Economic
Supplementary notes
Representation (EMPOWER) is an alliance
IN-SPACe of G20 business leaders and governments
 Indian National Space Promotion and that aims to accelerate women’s leadership
Authorisation Centre (IN-SPACe) is a and empowerment in the private sector.
O

single-window, independent, nodal agency The G20 EMPOWER 2023 under India’s
C


S.

that functions as an autonomous agency in presidency aims to take forward India’s


TE

the Department of Space (DOS). women-led development agenda.


O
FN

PPP-PTS-4392/082023/17 19
D
.P
W
W
W
https://www.pdfnotes.co/

Why in News? Supplementary notes


 The G20 EMPOWER Summit had a total Infrastructure Investment Trust (InvITs)
participation of over 300 participants (263
 An Infrastructure Investment Trust
domestic participants and 70 international
(InvITs) is Collective Investment Scheme
delegates).
similar to a mutual fund, which enables
 The international delegates were from direct investment of money from individual
13 G20 and 4 guest countries and 8 and institutional investors in infrastructure
International Organisations. projects to earn a small portion of the income
as return.
 These included Argentina, Australia,
Brazil, France, Indonesia, Italy, Japan,  The InvIT is designed as a tiered structure
Mexico, Republic of Korea, Saudi Arabia, with Sponsor setting up the InvIT which in
South Africa, UK, USA, Bangladesh, The turn invests into the eligible infrastructure
Netherlands, Spain, UAE, WTO, ISA, UN projects either directly or via special purpose
Women, UN, UNICEF, ILO, World Bank vehicles (SPVs).
and ADB.  The InvITs are regulated by the SEBI
(Infrastructure Investment Trusts)
Regulations, 2014.
53. Correct Option: (b)
Why in News?
Explanation:

RE
 After the success of the National Highways
 Statement 3 is incorrect: The technology Authority of India’s (NHAI’s) first
has been developed in-house by the UIDAI. infrastructure investment trust (InvIT)
with foreign institutional investors, the
Supplementary notes Centre is working on a proposal to launch
Finger Minutiae Record- Finger Image a fresh InvIT for national highways, where
Record (FMR-FIR) modality domestic retail investors can hold units of
the trust.
O
 As more frauds involving the Aadhaar-
enabled Payment System (AePS) come to
light, the Unique Identification Authority 55. Correct Option: (b)
of India (UIDAI) has turned to artificial
Explanation:
intelligence-based systems in an effort to
SC
limit the cases, including the development  Statement 3 is incorrect: Blindness from
of technologies based on fingerprinting and trachoma is irreversible.
facial recognition. Supplementary notes
 Recently it has been told that the UIDAI
Trachoma
has rolled out an in-house Artificial
Intelligence/Machine Learning  Trachoma is the leading infectious cause
technology-based Finger Minutiae of blindness worldwide. It is caused by an
Record – Finger Image Record (FMR- obligate intracellular bacterium called
GS

FIR) modality that can check the liveness Chlamydia trachomatis.


of a fingerprint to detect the use of cloned  The infection is transmitted by direct or
fingerprint during the authentication indirect transfer of eye and nose discharges
process to prevent AePS frauds by the use of infected people, particularly young
of spoofed fingerprints during Aadhaar children who harbour the principal reservoir
authentication. of infection. These discharges can be spread
by particular species of flies.
Why in News?
 Trachoma is hyperendemic in many of the
 The technology uses a combination of both poorest and most rural areas of Africa,
finger minutiae and finger image to check Central and South America, Asia, Australia
the liveness of the fingerprint captured. The and the Middle East. It is responsible for
measure was implemented after instances the blindness or visual impairment of about
of people creating fake fingerprints 1.9 million people. It causes about 1.4% of
using silicone to syphon off money from all blindness worldwide.
unsuspecting individuals’ bank accounts
Why in News?
were reported.
 Iraq has now joined the league of 17 other
countries that have eliminated trachoma, a
54. Correct Option: (d) neglected tropical disease and the world’s
O

leading infectious cause of blindness,


C

Explanation:
S.

the World Health Organization (WHO)


TE

 All statements are correct announced recently.


O
FN

20 PPP-PTS-4392/082023/17
D
.P
W
W
W
https://www.pdfnotes.co/

56. Correct Option: (c) 58. Correct Option: (b)


Explanation: Explanation:
 All statements are correct  Option (b) is correct: The Lunar Codex
initiative’s purpose is to remind future
Supplementary notes generations that war, pandemics, and
economic crises did not deter artistic
Himalayan vulture endeavors.
 The Himalayan vulture (Gyps himalayensis)
Supplementary notes
or Himalayan griffon vulture is an Old
World vulture native to the Himalayas and Lunar Codex initiative
the adjoining Tibetan Plateau.  Lunar Codex is stored on memory cards or
 It is one of the two largest Old World laser etched on NanoFiche, a 21st-century
vultures and true raptors. update on film-based microfiche. These will
ensure that the art forms reach the lunar
 It is listed as Near Threatened on the IUCN surface safely.
Red List.
 This initiative is a message to future
 The Himalayan Griffon Vulture feeds only generations to remind them that war,
on carrion. They gather around carcasses pandemics and economic crises did not stop
located by soaring and gliding over large people from creating works of art.

RE
areas.  The collection of art is gathered from 30,000
 Why in News? artists, writers, filmmakers and musicians
from 157 countries. The art forms include
 Himalayan vulture bred in captivity for the images, magazines, books, podcasts, movies
first time in India. and music, which are divided into four
capsules.
57. Correct Option: (b)
O  The first such capsule is known as the Orion
collection, which has already flown around
Explanation: the moon when it launched on the Orion
 Statement 3 is incorrect: According to spacecraft as part of NASA’s Artemis 1
mission last year. In the coming months, a
the IUCN Red List, the yellow-bellied sea
SC
series of lunar landers will take the Lunar
snake is a species of Least Concern.
Codex capsules to various destinations in
Supplementary notes craters at the moon’s South Pole and a lunar
plain called Sinus Viscositatis.
Yellow Belly Snake
Why in News?
 The scientific name of the yellow belly
 The collection of art is gathered from 30,000
is ‘Pelamis platurus’. The side of the tail
artists, writers, film-makers and musicians
is like the paddle of a boat and the lower
GS

from 157 countries. The art forms include


abdomen is yellow making it different from images, magazines, books, podcasts, movies
other reptile species. and music, which are divided into four
 The rare yellow belly snake, mainly found in capsules.
the Arabian Sea, is very poisonous and even
antivenom does not work for snake bites. 59. Correct Option: (a)
 The intensity of the poison according to the
Explanation
experts is so high that the person can die
due to the snake bite.  Statement 2 is incorrect: The privileges
are absolute unlike fundamental rights for
 Experts believe that once a yellow-belly the citizens.
snake bites, the human body will become
completely paralyzed. Supplementary notes
 There are 24 types of sea snakes in India Parliamentary Privileges
and the yellow belly found in Digha is the  Article 105 and Article 194 grant privileges
most poisonous. or advantages to the members of the
parliament so that they can perform their
Why in News? duties or can function properly without any
O

 A rare yellow belly snake was seen on the hindrances.


C
S.

coast of Digha Bay of Bengal amid huge  Such privileges are granted as they are
TE

tides recently. needed for democratic functioning.


O
FN

PPP-PTS-4392/082023/17 21
D
.P
W
W
W
https://www.pdfnotes.co/

 These powers, privileges and immunities  It’s called an Einstein cross – when the
should be defined by the law from time-to- curvature of space-time around a massive
time. foreground object splits the light behind
 These privileges are considered as special it into four, like the points of a cross. Its
provisions and have an overriding effect in confirmation and analysis adds to a slowly
conflict. growing catalog of these rare alignments,
which can help us better understand the
Privileges mentioned in the constitution more distant reaches of the Universe.
 Freedom of speech and publication under Why in News?
parliamentary authority
 Recently, An Einstein Cross spotted in space-
 Freedom from being arrested ‘Flower with four petals’. An “Einstein cross”
 Right to exclude strangers from its is a fascinating phenomenon that resembles
proceedings and hold secret sessions a flower with four petals.
 Right to prohibit the publication of its
reporters and proceedings 61. Correct Option: (c)
 Right to regulate internal proceedings
Explanation:
 Right to punish members or outsiders for
contempt  Option (c) is correct: The commissions on
the Official Language and a Committee of

RE
Why in News? Parliament on Official Language enshrined
 Rajya Sabha Chairman Jagdeep Dhankhar under Article 344 of constitution.
referred complaints related to the privilege
Supplementary notes
of the House against TMC’s Derek O’Brien
and AAP’s Raghav Chadha to the privileges Official Language
committee.
 The Committee of Parliament on Official
Language was constituted in 1976 under
O
60. Correct Option: (c) section 4 of the Official Languages Act,
1963.
Explanation:
 Article 344 (4) provides for formation of such
 Option (c) is correct: Einstein Cross is a a committee. It is a high powered committee
remarkable example of gravitational lensing
SC
which reviews the progress made in the use
in astronomy. It refers to the formation of of Hindi in official work of the Union.
multiple, closely spaced images of a single
distant quasar as a result of the gravitational  It periodically reviews the progress in the
bending of light by an intervening galaxy. use of Hindi and submits a report to the
President.
Supplementary notes
Why in News?
Einstein Cross  Recently, Home Minister of India
Gravitational lensing is one of astronomy’s
GS

 emphasised the importance of respecting


great wonders: a natural lens that heritage and erasing signs of slavery by
magnifies the distant universe. Sometimes promoting Indian languages, including the
a lensing system takes the shape of a so- official language.
called “Einstein Cross”. Those are rare and
amazingly useful ways to study objects far
away in space and time. 62. Correct Option: (a)
 A team of astronomers recently found a new Explanation:
one using the Dark Energy Spectroscopic
Instrument mounted on a telescope at Kitt  Both statements are correct
Peak National Observatory. Supplementary notes
 This instrument is surveying the sky and
Bhu-Vision (Krishi-RASTAA)
has found many instances of gravitational
lensing.  The machine can test the sample for 12
parameters and send a soil health card to
 Followup observations show the new one to
mobile phones in flat 30 minutes.
be both beautiful and a scientific treasure
trove of information about the early  The Bhu-Vision platform, also known as the
universe. Krishi-Rasta soil testing equipment, was
A massive galaxy has created a rare launched here by RM Sundaram, director
O


of the Indian Institute of Rice Research
C

distortion in the path of light that has


S.

traveled billions of years to reach us from a (IIRR), a division of the Indian Council of
TE

more distant galaxy. Agricultural Research. Thursday.


O
FN

22 PPP-PTS-4392/082023/17
D
.P
W
W
W
https://www.pdfnotes.co/

 Bhu-Vision (Krishi-RASTAA) is jointly Supplementary notes


developed by ICAR-IIRR and Krishitantra,
manufactured by HiMedia Laboratories Central Registrar of Cooperative
and marketed by BhoomiSeva (HCF Tech Societies
Services Private Limited).  As per the Constitution, the Cooperative
societies with objects confined to one State
Why in News?
only are governed by the Cooperative laws
 In a significant development for agricultural of the respective State Government and the
technology, a revolutionary IoT-based cooperative societies with objects confined
automated soil testing and agronomy to more than one State are governed by
advisory platform, Bhu-Vision (also known the central law, namely, ‘the Multi-State
as KRISHI-RASTAA Soil Testing System) Co-operative Societies Act 2002 (Act 39 of
was officially launched at AICRP (ICAR- 2002).
IIRR), Hyderabad.
 The Central Registrar of Cooperative
Societies is appointed as per article 243ZH(f)
63. Correct Option: (a) of the Constitution read with subsection (1)
of section (4) of Multi-State Co-operative
Explanation:
Societies Act 2002 and is the statutory
 Statement 2 is incorrect: It uses both body responsible for registration and other

RE
electro-optical and fibre-optic technologies. processes of the Multi State Cooperative
Supplementary notes Societies (MSCS).

Spike non-line of sight (NLOS) Why in News?


 Spike is a family of anti-tank missiles made  The digital portal of the Central Registrar of
by Rafael Advanced Defense Systems, an Cooperative Societies (CRCS) office in Pune,
Maharashtra has been launched by Union

Israeli state-run company.
O
These missiles can strike targets at a
Home Minister and Minister of Cooperation
Minister.
maximum range of 50 km when launched
from helicopters.
65. Correct Option: (c)
SC
 The missile comes with a real-time wireless
data link and uses its seeker and wireless
Explanation
data-link to give the user real-time video
imagery and man-in-the-loop control  All statements are correct
throughout its flight.
Supplementary notes
 When operated in mid-course navigation
mode, this feature will allow the user to Digital India RISC-V (DIR-V) Program
control the missile in flight to “alter or The Digital India RISC-V (DIR-V)
GS


abort the mission” while it is en route to the Program is to create the next generation of
target. indigenous microprocessors as a part of its
“Atmanirbhar Bharat” initiative.
Why in News?
 The program aims to achieve heavy-
 Indian Air Force (IAF) has received the
grade commercial silicon and design wins
Israeli Spike non-line of sight (NLOS) long
range anti-tank missile. by December 2023, implying that the
indigenous chipsets are expected to enter
mass production.
64. Correct Option: (b)  Design win is a technical term used in the
Explanation: semiconductor industry to indicate that a
company’s chip/product has been designed
 Statement 3 is incorrect: The Cooperative
into a fairly large design that will result in a
societies with objects confined to one State
lot of ongoing, volume sales of the chip.
only are governed by the Cooperative laws
of the respective State Government and the  The federal government has set a timeline
cooperative societies with objects confined to commercially roll out the first indigenous
to more than one State are governed by chipsets – Shakti and Vega – by 2023-24 to
O

the central law, namely, ‘the Multi-State meet the surging demand for semiconductors
C
S.

Co-operative Societies Act 2002 (Act 39 of in the automotive, mobility, and computing
TE

2002). segments.
O
FN

PPP-PTS-4392/082023/17 23
D
.P
W
W
W
https://www.pdfnotes.co/

Why in News? Supplementary notes


 Addressing the Digital India RISC-V Compulsory Convertible Debenture
Symposium on ‘future of Electronics in (CCD)
India through the RISC-V pathway in a
 A compulsory convertible debenture (CCD)
virtual mode, the Centre is committed to is a type of bond which must be converted
making DIR-V the Indian Instruction Set into stock by a specified date. It is classified
Architecture. as a hybrid security, as it is neither purely a
bond nor purely a stock.
66. Correct Option: (b)  A debenture is a medium- to long-term debt
security issued by a company as a means of
Explanation: borrowing money at a fixed interest rate.
 Statement 2 is incorrect: With a SaaS Unlike most investment-grade corporate
model, the software is hosted on remote bonds, it is not secured by collateral. It is
backed only by the full faith and credit of
servers, maintained and updated by the
the issuing company.
service provider, and made available to
customers via web browsers, mobile apps Why in News?
and APIs.  Notices issued to those invested in fully or
Supplementary notes compulsorily convertible debentures issued

RE
by Indian companies.
Software-as-a-Service (SaaS)
 It is a cloud based software delivery model 68. Correct Option: (c)
that allows end users to access software
applications over the internet. Explanation:
 With a SaaS model, the software is hosted  All statements are correct
O
on remote servers, maintained and updated Supplementary notes
by the service provider, and made available
to customers via web browsers, mobile apps Chief Election Commissioner (CEC)
and APIs.  The Election Commission of India is an
autonomous constitutional authority
SC
 SaaS provides a number of user benefits
responsible for administering Union and
over traditional software delivery models,
State election processes in India.
including reduced upfront costs, scalability,
flexibility, and accessibility.  The body administers elections to the Lok
Sabha, Rajya Sabha, State Legislative
 As the software is hosted on the service Assemblies in India, and the offices of
provider’s servers, there is no need for users the President and Vice President in the
to invest in expensive infrastructure in country.
order to use the software.
GS

 The chief election commissioner is the chief


 Instead, the end users pay a subscription of the election commission of India
fee to access the software as on-demand  The chief election commissioner has the
service. power over all the elections in the country
Why in News? involving the president, the vice president,
the state legislatures, the MPs, the MLAs,
 India’s SaaS industry to reach $50 billion etc.
by 2030, to generate revenue of $20-25
million. Why in News?
 With the view of overturning the effect of the
Supreme Court verdict on the appointment
67. Correct Option: (a) of the Chief Election Commissioner (CEC)
and Election Commissioners (ECs), a Bill
Explanation
was listed to be introduced in Rajya Sabha.
 Statement 2 is incorrect: For companies,
it allows for repayment of debt without
spending cash. 69. Correct Option: (b)
 Statement 3 is incorrect: A compulsory Explanation:
O

convertible debenture (CCD) is classified as Statement 3 is incorrect: It occurs in high-


C


S.

a hybrid security, as it is neither purely a altitude regions like hills and mountains
TE

bond nor purely a stock. due to the formation of low-pressure areas.


O
FN

24 PPP-PTS-4392/082023/17
D
.P
W
W
W
https://www.pdfnotes.co/

Supplementary notes Supplementary notes


Cloudburst HeLa cells
 A cloudburst is defined as an intense  HeLa cells are a type of immortal human
downpour that lasts only a brief time and cell line that were the first to be successfully
is occasionally accompanied by hail and cultured and continuously propagated
thunder. outside the human body.
 These cells were derived from a cervical
 It is defined by the India Meteorological
cancer biopsy taken from Henrietta Lacks,
Department (IMD) as unexpected
an African-American woman, in 1951. The
precipitation that exceeds 100mm (or 10 name “HeLa” is derived from the first two
cm) per hour across a region of land that is letters of her first and last names.
between 20 and 30 square kilometres.
 HeLa cells have been widely used in
Why in News? scientific research, including cell biology,
genetics, virology, and drug testing.
 Cloudbursts are a common occurrence in
Himalayan regions but experts are alarmed  They have contributed to a multitude
by the increase in extreme weather-related of medical advancements, such as
events. the development of the polio vaccine,
understanding cancer, and various other
scientific breakthroughs.

RE
70. Correct Option: (d)  HeLa cells are known for their ability to
divide and multiply rapidly, making them
Explanation: an invaluable resource in laboratories
 Statement 2 is incorrect: Deflation can worldwide.
be caused by an increase in productivity, a
Why in News?
decrease in overall demand, or a decrease in
the volume of credit in the economy.  The estate of Henrietta Lacks filed a lawsuit

Supplementary notes
O in Maryland federal court on Thursday
accusing biopharmaceutical company
Ultragenyx Pharmaceutical (RARE.O) of
Deflation unlawfully profiting from cells that were
Deflation is when the general price levels taken from Lacks’ body without her consent
SC

in a country are falling—as opposed to during a medical procedure in 1951.
inflation when prices rise.
 Deflation can be caused by an increase in 72. Correct Option: (b)
productivity, a decrease in overall demand,
Explanation:
or a decrease in the volume of credit in the
economy.  Statement 3 is incorrect: Any community
or private land may be designated by the
 Most of the time, deflation is unambiguously State Government as a Community Reserve
GS

a positive trend for the economy, but it can provided the inhabitants of that community
also under certain conditions occur along or the person concerned consent to provide
with a contraction in the economy. such areas for the preservation of the local
 In an economy dominated by debt-fueled flora and fauna as well as their customs,
asset price bubbles, deflation can lead to a cultures, and practises.
temporary financial crisis and a period of Supplementary notes
liquidation of speculative investment known
as debt deflation. Community Reserves
 Community reserves fall under protected
Why in News? areas, along with marine protected areas,
 China reported “deflation” in July. national parks, wildlife sanctuaries and
conservation reserves, according to the Wild
Life (Protection) Act (WLPA), 1972.
71. Correct Option: (c)  Conservation reserves and community
Explanation: reserves are protected areas of India which
typically act as buffer zones between
 Option (c) is correct: HeLa cells are a established national parks, wildlife
type of immortal human cell line that were sanctuaries and reserved and protected
O

the first to be successfully cultured and forests of India, according to the Union
C
S.

continuously propagated outside the human Ministry of Environment, Forests and


TE

body. Climate change (MoEFCC).


O
FN

PPP-PTS-4392/082023/17 25
D
.P
W
W
W
https://www.pdfnotes.co/

 These protected area categories were first  A quota formula is used to help assess
introduced in the Wild Life (Protection) members’ relative position in the world
Amendment Act of 2002 — the amendment economy and it can play a role in guiding
to WLPA, 1972. the distribution of quota increases. The
current formula was agreed to in 2008.
 The provisions of the WLPA apply to an
area once it has been declared a community Why in News?
reserve. Section 33 of the WLPA passes
 Reserve Bank Governor Shaktikanta Das
the authority of the sanctuary to the chief
pitched for the “expeditious completion” of
wildlife warden. the 16th general review of the quotas at
Why in News? the International Monetary Fund (IMF),
pointing out that the same can help the
 In a written reply, Rajya Sabha has been multilateral lender assist distressed
told that institutions such as traditional countries in a better way.
village durbars in Meghalaya play a vital
role in Meghalaya’s society and economy,
inter alia environmental conservation 74. Correct Option: (c)
and raising awareness about sustainable
practices. Government of Meghalaya has Explanation:
already notified several sacred forests  Both statements are correct
as Community Reserves under Wildlife
(Protection) Act, 1972. Supplementary notes:

RE
Inclusive Conservation Initiative (ICI)
73. Correct Option: (b)  The Inclusive Conservation Initiative (ICI)’s
goal is to enhance Indigenous Peoples’ and
Explanation: Local Communities’ (IPs and LCs) efforts to
 Statement 3 is incorrect: Quotas are steward land, waters and natural resources
denominated in Special Drawing Rights to deliver global environmental benefits.
(SDRs), the IMF’s unit of account. It is funded by the Global Environment
O

Facility (GEF) and supported by Conservation
Supplementary notes International and the International Union
IMF Quotas for Conservation of Nature (IUCN).
 The IMF Quotas are the building blocks  The Inclusive Conservation Initiative (ICI)
SC
of the IMF’s financial and governance supports IP and LC-designed and -led
structure. An individual member country’s activities that protect biodiversity and result
quota broadly reflects its relative position in in other global environmental benefits.
the world economy. Quotas are denominated  Supervision and technical guidance are
in Special Drawing Rights (SDRs), the IMF’s provided by Conservation International
unit of account. (CI) and the International Union for the
Conservation of Nature (IUCN), serving as
 The IMF’s Board of Governors conducts
joint GEF Implementing Agencies.
general quota reviews quotas at least every
GS

five years. The two main issues addressed in  Both organizations bring their decades of
a general review of quotas are the size of an collective experience working with IPs and
overall quota increase and the distribution LCs, as well as their regional and global
of the increase among the members. expertise, to the implementation of the GEF
Inclusive Conservation Initiative.
 First, a general quota review allows the
IMF to assess the adequacy of quotas both Why in News?
in terms of members’ balance of payments  Even though the volume of commitments
financing needs and in terms of the IMF’s of funding for Indigenous Peoples and local
own ability to help meet those needs. Second, communities (IP and LC) in improving
a general review allows for increases in conservation has increased in recent years,
members’ quota to reflect changes in their the funding landscape has not changed
relative positions in the world economy. In much for them.
addition, a member may request an ad hoc
 A new report summarising the first
quota adjustment at any time outside of a
phase of implementation of the Inclusive
general review.
Conservation Initiative (ICI), endorsed in
 If the Board of Governors determines that an January 2022, highlights that while there
increase is needed, it considers the amount is mounting evidence on the potential of
of increase overall and how to distribute it Indigenous Peoples and local communities
among members. Any changes in quotas in improving conservation, in total, donors
O

requires approval by 85% of the total voting have directed less than 1 per cent of climate
C
S.

power and a member’s own quota cannot be change mitigation and adaptation funding
TE

changed without its consent. to IPs and LCs.


O
FN

26 PPP-PTS-4392/082023/17
D
.P
W
W
W
https://www.pdfnotes.co/

75. Correct Option: (c)  While this can occur in home settings, it
is more common in large buildings, for
Explanation: example hotels, cruise ships or hospitals.
 Option (c) is correct  If people inhale small droplets of water
containing the bacteria, they can contract
Supplementary notes
legionnaires’ disease, a type of severe
Consumer Welfare Fund (CWF) pneumonia. While this can be treated with
antibiotics, it can lead to lung failure and
 CWF has been constituted under Section 57
even death.
of the Central Goods & Service Tax (CGST)
Act, 2017.  Symptoms include a cough, shortness
of breath, a high temperature, flu-like
 The provision for utilization of CWF has
symptoms and chest pain or discomfort.
been made in Section 58 of the CGST Act,
People with such symptoms are advised
2017 which provides that the fund shall be
to contact a medical professional, and tell
utilized by the Government for the welfare
them if they have been staying in a setting
of the consumers in such manner as may be
such as a hotel, spa or hospital in the past
prescribed.
10 days.
 The overall objective of the Consumer
 Legionella bacteria can also cause Pontiac
Welfare Fund is publicity or consumer
fever, a mild flu-like illness. Experts say it
awareness on Goods and Services Tax (GST)

RE
is not clear why some people get this less
which may include providing financial
severe illness while others get legionnaires’
assistance and to implement measures
disease when exposed to the same bacteria.
to promote and protect the welfare of the
consumers of goods and services in so far, Why in News?
they pay or bear the burden of GST.  Deadly bacteria outbreak forces all migrants
 The Fund would strengthen the consumer off Bibby Stockholm barge after four days.
by enhancing their awareness about the
O
provisions of GST and more specifically
77. Correct Option: (b)
about their rights & responsibilities under
GST. Explanation
Why in News? Statement 3 is incorrect: There is no
SC

 Consumer Welfare Fund (CWF) aims to cure.
support government initiatives to enhance Supplementary notes
consumer awareness about GST.
Type 2 Battens Disease
Battens Disease refers to a group of genetic
76. Correct Option: (b) 
disorders classified as neuronal ceroid
Explanation: lipofuscinoses (NCLs). NCLs, which are
GS

rare and fatal, can affect adults as well as


 Statement 1 is correct: Legionella bacteria
children. There are 13 NCLs under Batten
tend to cause problems in water systems
disease. They are classified under the
with suitable temperature conditions for
following parameters:
their proliferation.
 Onset age
 Statement 2 is correct: Legionnaires’
disease is a type of severe pneumonia  Symptom
contracted by inhaling small droplets of  Severity
water containing the bacteria.
 According to the Cleveland Clinic, Battens
 Statement 3 is incorrect: Pontiac fever is Disease is passed down from parents to
a mild flu-like illness caused by legionella children. It is caused by a faulty gene
bacteria, not a more severe illness. that impacts the body’s ability to properly
break down and dispose of cellular waste.
Supplementary notes
This causes the body to stop functioning
Legionella Bacteria properly.
 The legionella bacteria occur in rivers and  While symptoms vary, the first signs of it
ponds, but only tend to cause problems in are -
water systems where the temperature is  Vision loss (not in adults)
O

right for them to proliferate. Such settings


C

 Seizures
S.

can include hot tubs, showers and air


TE

conditioners.  Cognitive issues


O
FN

PPP-PTS-4392/082023/17 27
D
.P
W
W
W
https://www.pdfnotes.co/

 Trouble speaking India Movement. Barua, being halted by


 Coordination problems the station-in-charge, told him she would do
her duty and he should do his.
 In the United States, of every 100,000
children, around 2-4 will be affected by this  British forces at the station then opened
condition.Unfortunately, there is no cure indiscriminate fire on the group. Barua,
for Battens Disease. who was at the forefront of the procession,
was shot dead at point-blank range.
Why in news:
Why in News?
 Type 2 Battens Disease, also known as
childhood dementia, is a genetic disorder  President Droupadi Murmu in her speech
that affects around 2 to 4 in every 100,000 on the eve of Independence Day hailed the
children. Bryden and Kristian Tucker, the contribution of women freedom fighters.
parents of Michael (6) and Oliver (2), say
they are devasted but trying to make the
best possible life for their children.
79. Correct Option: (c)
Explanation:
78. Correct Option: (b)  All statements are correct

Explanation: Supplementary notes

RE
 Option (b) is correct: Kanaklata Barua Pradhan Mantri Uchchatar Shiksha
was born in Barangabari, Assam in 1924. Abhiyan
On 20 September, 1942 she joined a group
of freedom fighters and marched towards  Launched in 2013, the PM-USHA aims at
Gohpur Police station to hoist the Tricolour providing strategic funding to eligible state
in support of the Quit India Movement. higher educational institutions.
Police at the station opened indiscriminate  The central funding is based on norms and
fire on the group to prevent the team from is outcome dependent.
O
committing such a daring act. Kanaklata
 Funds flow from the central ministry
Barua was martyred at a young age with a
through the state governments/union
Tricolour furling in her hands.
territories before reaching the identified
Supplementary notes institutions.
SC

Kanaklata Barua  Funding to states would be made on the


basis of the critical appraisal of State Higher
 Kanaklata Barua was born on 22 December
Education Plans, which would enlist each
1924 in Assam’s Barangabari village. Her
state’s strategy to address issues of equity,
parents were Krishna Kanta Barua and
access and excellence in higher education.
Korneswari Barua. Barua’s mother passed
away at age five. Her father remarried, but  PM-USHA places greater emphasis on the
would pass away less than a decade later. improvement of the quality of teaching-
GS

 These developments came in the backdrop learning processes in order to produce


of the freedom struggle finding more and employable and competitive graduates,
more backing in Assam – particularly post-graduates and PhDs.
among women. Many of these women joined  Spread across two plan periods (XII and
‘Shanti Bahini’ and the ‘Mrityu Bahini.’ XIII), the programme focuses on state
 While the members of the former worked higher educational institutions and draws
peacefully, members of the latter group upon the best practices from colleges and
readied themselves to face death. Barua universities across the nation.
initially wanted to join Netaji Subhas Why in News?
Chandra Bose’s Azad Hind Fauj.
 PM-USHA, a centrally sponsored
 However, her status as a minor excluded her
programme, aims to work with 300-plus
from joining. Barua then joined the Mrityu
state universities and its affiliated colleges.
Bahini group. Though the Mrityu Bahini
had a similar policy of not allowing minors,
Barua managed to persuade them to allow 80. Correct Option: (c)
her to join and was even put in charge of
the cadre. Explanation
 On 20 September, 1942, Barua, aged just  Statement 3 is incorrect: NAFIS assigns
O

17, led a group of 5,000 freedom fighters on a unique 10-digit National Fingerprint
C
S.

a march towards Gohpur Police station to Number (NFN) to each person arrested for
TE

hoist the Tricolour in support of the Quit a crime.


O
FN

28 PPP-PTS-4392/082023/17
D
.P
W
W
W
https://www.pdfnotes.co/

Supplementary notes 82. Correct Option: (b)


 The web-based application functions Explanation:
as a central information repository by
consolidating fingerprint data from all  Statement 3 is incorrect: In 1926,
states and Union Territories. Swami Shraddhanand wrote a book Hindu-
Sangathan (Savior of the Dying Race). That
 According to a 2020 report by the NCRB, it book played a key role in the revival of
enables law enforcement agencies to upload, Hinduism.
trace, and retrieve data from the database
in real time on a 24×7 basis. Supplementary notes
 NAFIS assigns a unique 10-digit National Maharishi Dayanand Saraswati
Fingerprint Number (NFN) to each person
 Maharishi Dayanand Saraswati was born
arrested for a crime.
on February 12, 1824 in Tankara (Gujarat).
 This unique ID will be used for the person’s His father’s name was Karsan Ji Tiwari and
lifetime, and different crimes registered mother’s name was Amritbai.
under different FIRs will be linked to the
 Born in a Brahmin clan, Mool Shankar
same NFN.
attained enlightenment on the occasion of
 The 2020 report states that the ID’s first two fasting on the day of Shivratri. He suddenly
digits will be that of the state code in which left home at the age of 19 and went on a
the person arrested for a crime is registered, remarkable journey in search of the true

RE
followed by a sequence number. Shiva (God).
Why in News?  He received education from Guru
VirjanandJi in Mathura and understood
 Conceptualized and managed by the NCRB
the basic thing that in the present state of
at the Central Fingerprint Bureau (CFPB)
the country, the root cause of misery is the
in New Delhi, the National Automated
prevalence of wrong facts in place of correct
Fingerprints Identification System (NAFIS)
meaning of Vedas.
O
project is a country-wide searchable
database of crime- and criminal-related  Because of that, the trap of religious pomp,
fingerprints. superstition, social evils has spread in the
society and despite being our own country,
we are slaves of others.
81. Correct Option: (c)
SC
 Seeing the country’s poverty, illiteracy,
Explanation: plight of women, destruction of language
and culture, Dayanand was deeply moved.
 All statements are correct After traveling in the Himalayas for some
Supplementary notes time and with penance, he participated
inKumbhMela in Haridwar.
Metagenome Sequencing
 He made his point with strength and logic.
 Metagenomics is the study of the entire He tried to establish the truth by strongly
GS

genetic content of all microbiota members debating with the opponents.


in a natural habitat by utilisation of the
 He had only one goal in life - Must know
whole genome sequencing technique. In
about true and then believe it. He also gave a
contrast, metataxonomics utilises only 16S
motivational slogan that one should always
rRNA analysis.
be ready to accept the truth and leave the
 Metagenomics is the study of the structure untruth. He composed a classic book named
and function of entire nucleotide sequences Satyarth Prakash to express his point of
isolated and analyzed from all the organisms view on religious, social, political subjects.
(typically microbes) in a bulk sample.
 He toured the whole country, Punjab,
 Metagenomics is often used to study a Bengal, Maharashtra and especially Uttar
specific community of microorganisms, such Pradesh was his work area.
as those residing on human skin, in the soil
 He created this awareness for the first
or in a water sample.
time in the context of cow protection, Hindi
Why in News? protection, utility of Sanskrit language,
Swadesh, Swadeshi, Swabhasha, and self-
 With SARS-CoV-2, scientists didn’t go the respect.
more time-consuming microbiology route;
instead, in a break from tradition, the Why in News?
O

samples were directly subjected to genome- Knowing that life is not immortal, he
C


sequencing and bioinformatic analysis, an
S.

established Arya Samaj in Mumbai in 1875


approach called metagenomics.
TE

for continuous expansion of his work.


O
FN

PPP-PTS-4392/082023/17 29
D
.P
W
W
W
https://www.pdfnotes.co/

83. Solution: (c) with South Korea, but the trilateral format
is not a triple alliance structure. Rather,
Explanation: it is a strategic partnership among the
 Both statements are correct three countries, a defensive arrangement
responding to an array of challenges from
Supplementary notes both China and North Korea.
Promotion of Research & Innovation in  “The Spirit of Camp David” statement issued
Pharma-MedTech (PRIP) by the three leaders is a political agreement,
 In a move to strengthen the research and rather than a legal one requiring signature,
development (R&D) and innovation efforts but it nevertheless is a powerful document.
of Indian pharma industry and academia, It represents a major effort to establish
the Cabinet has approved the National precedent, create momentum, and set the
Policy on Research & Development and course for trilateral security cooperation
Innovation in the Pharma-MedTech Sector among the three countries.
and the Scheme for Promotion of Research  In democracies, administrations come and
and Innovation in Pharma-MedTech go; with this statement, the three leaders
(PRIP). are attempting to establish a legacy that will
 The objective of the scheme is to transform continue irrespective of political changes in
the Indian Pharma MedTech sector from their respective capitals.

RE
cost-based to innovation-based growth by
 They are doing this because the fundamental
strengthening the research infrastructure
national security interests of the three
in the country.
countries and the consequent impetus
 Rs 5,000 crore will be spent in five years to strengthen cooperation among them
under the PRIP scheme, and companies transcend the political vagaries of any given
will be given incentives for research. The administration.
scheme is being started for the year 2023-
24 to 2027-28. Why in News?
O
 Under the scheme, seven centers of  US President Joe Biden hosted the first-
excellence will be opened for research in the ever trilateral summit bringing together
pharma sector. the leaders of the United States, Japan, and
South Korea at Camp David in Maryland.
SC
Why in News?
 New centers will be opened in the existing
seven National Institutes of Pharmaceutical 85. Correct Option: (b)
Education & Research (NIPERs).
Explanation:
 Statement 2 is incorrect: IDF-NBFCs
84. Correct Option: (a) are allowed to raise funds through rupee or
Explanation: dollar-denominated bonds with a minimum
GS

maturity of five years.


 Option: (a) is correct: US President Joe
Biden hosted the first-ever trilateral summit Supplementary notes
bringing together the leaders of the United
States, Japan, and South Korea at Camp Infrastructure Debt Funds - Non-Banking
David in Maryland. The summit resulted Financial Companies (IDF-NBFCs)
in a joint statement dubbed “The Spirit of  An IDF-NBFC refers to a non-deposit-
Camp David”. taking NBFC that is authorised to refinance
Supplementary notes infrastructure projects that have completed
at least one year of satisfactory commercial
 The summit resulted in a joint statement
operations.
dubbed “The Spirit of Camp David”. This
is the first stand-alone trilateral summit  Additionally, IDF-NBFCs can directly
among the three countries, and the first finance toll-operate-transfer (TOT)
Camp David summit hosted by Biden during projects.
his presidency, both of which are intended  To qualify as an IDF-NBFC, entities must
to imbue the summit with special historic adhere to specific net owned funds (NOF)
significance and symbolism.
and regulatory capital requirements.
 Irrespective of Chinese and North Korean
Why in News?
O

posturing, this does not represent the


C

kernel of a “NATO in Asia.” The United RBI Revises Guidelines for Infrastructure
S.


States has an alliance with Japan, and one
TE

Debt Fund-NBFCs.
O
FN

30 PPP-PTS-4392/082023/17
D
.P
W
W
W
https://www.pdfnotes.co/

86. Correct Option: (b)  The agreement covers various sectors,


including agriculture, manufacturing,
Explanation: textiles, chemicals, machinery, and more.
 Statement 1 is incorrect: The species Why in News?
is found in dense riverine forests in the
high valleys and mountains of Jammu and  India and the Association of Southeast
Kashmir and northern Himachal Pradesh. Asian Nations (ASEAN) have agreed to
review their trade pact by 2025. The Indian
Supplementary notes government said recently less than a month
after India’s trade minister called the
Kashmir stag (Cervus hanglu hanglu)
agreement unfair to the Indian industry.
 The Kashmir stag (Cervus hanglu hanglu),
also called hangul, is a subspecies of
Central Asian red deer endemic to Kashmir 88. Correct Option: (c)
and surrounding areas. It is found in dense
riverine forests in the high valleys and
Explanation:
mountains of Jammu and Kashmir and  Option (c) is correct: Lunar dust,
northern Himachal Pradesh. composed of minute abrasive particles from
the lunar surface, emerges as a pivotal
 Hangul Cervus hanglu is the only red deer
adversary during soft landings.
species in India distributed in the Kashmir
Himalaya. Hangul is a critically-endangered

RE
Supplementary notes
species as per the International Union
for Conservation of Nature and Natural Moon Mission
Resources (IUCN) Red List of IUCN, Red  The region of the Moon’s South Pole has
List Data, a Schedule I Species in Indian a difficult terrain, full of craters and deep
Wildlife Protection Act 1972. trenches. It is also far from the equatorial
 The species is under threat based on the region explored by previous lunar missions.
vulnerability of population with regard Some areas on the South Pole are shrouded
O
to viability and structure, dwindling in darkness and have never received
population, and habitat vulnerability. sunlight.
 Temperatures are so cold there that they
Why in News?
can plummet to as low as -230 degree
SC
 After two years, the unique Kashmir stag, Celsius. This rocky terrain, complete
vernacularly known as Hangul, the state darkness and extremely cold weather make
animal of Jammu and Kashmir, has again it more difficult for electronic instruments
shown a marginal increase in its population, to function properly.
much to the delight of wildlife department
and Hangul lovers across the region. Why in News?
 India is waiting with bated breath for Moon
mission Chandrayaan-3’s successful landing
87. Correct Option: (c) on the lunar surface. The spacecraft’s lander
GS

Explanation: is set to attempt a soft landing on the Moon’s


South Pole.
 Both statements are correct.
Supplementary notes:
89. Correct Option: (b)
ASEAN-India Trade in Goods Agreement
Explanation:
(AITIGA)
 Statement 3 is incorrect: Unlike the
 The ASEAN-India Trade in Goods
Global NCAP, Bharat NCAP will not just
Agreement (AITIGA) was signed in 2009.
comprise a standardized test. Bharat NCAP
AITIGA is a free trade agreement (FTA)
will look to take a more comprehensive
between the Association of Southeast Asian
approach and will try to include all sorts of
Nations (ASEAN) and India, aimed at
factors pertaining to the Indian market.
promoting trade and economic cooperation
between the two regions. Supplementary notes
 ASEAN is a regional intergovernmental Bharat NCAP
organization comprising ten Southeast
Asian countries, while India is a South  The Bharat NCAP is India’s very own
Asian country. automobile safety performance assessment
O

programme in which vehicles sold in the


C

 AITIGA aims to reduce or eliminate tariffs domestic market will be tested and graded
S.

on a wide range of goods traded between on several measures.


TE

ASEAN member states and India.


O
FN

PPP-PTS-4392/082023/17 31
D
.P
W
W
W
https://www.pdfnotes.co/

 With the launch of this programme, India Why in News?


becomes the fifth country in the world –
 Commonwealth Parliamentary Association
behind the United States, China, Japan
partners with UN Women to promote
and South Korea – to have its dedicated car
gender equality through parliamentary
crash safety program.
engagement.
 The Bharat NCAP, the safety assessment
programme, will be applicable to all
passenger vehicles that can seat up to eight 91. Correct Option: (c)
people and weigh less than 3.5 tonnes,
starting from 1 October. Explanation:
 In addition to petrol and diesel vehicles,  All statements are correct.
Bharat NCAP could also offer tests and Supplementary notes
safety ratings for CNG and hybrid cars.
 As part of the programme, cars will be Astra Missile
crash-tested and given points which in turn  Indigenously developed, ASTRA is a state-
would translate into stars. The safest of the of-the-art BVR air-to-air missile to engage
lot will get five stars. and destroy highly manoeuvring supersonic
aerial targets.
Why in News?
 It is jointly designed and developed by
 India’s road and car safety just received

RE
the Defence Research and Development
mass acceleration as the government
launched the much-awaited Bharat New Laboratory (DRDL), Research Centre
Car Assessment Programme (Bharat Imarat (RCI) and other laboratories of
NCAP), India’s homegrown vehicle safety DRDO.
rating system.  The indigenous Astra BVR firing from
homegrown Tejas fighters is expected to
boost the ‘Aatmanirbhar Bharat’ (self-
90. Correct Option: (b)
O
reliant India) programme of the Narendra
Explanation: Modi government.
 Statement 3 is incorrect: The nine Regions Why in News?
of the CPA are: Africa; Asia; Australia;
 Indigenously developed Light Combat
British Islands and Mediterranean (BIM);
SC
Aircraft (LCA) Tejas successfully test-fired
Canada; Caribbean, Americas and the
Atlantic (CAA); India; Pacific; and South- an indigenously developed ASTRA beyond
East Asia. The CPA India Regional visual range (BVR) air-to-air missile off the
Secretariat is based within the Lok Sabha coast of Goa.
at the Parliament of India.
Supplementary notes 92. Correct Option: (a)
Regions of the Commonwealth Explanation:
GS

Parliamentary Association (CPA)  Statement 2 is incorrect: It is established


 The Commonwealth Parliamentary by the WHO and the Drugs for Neglected
Association (CPA) is one of the oldest Disease initiative (DNDi).
established organisations in the
Commonwealth. Supplementary notes

 Founded in 1911, it is a membership Global Antibiotic Research and


association which brings together Members, Development Partnership (GARDP)
irrespective of gender, race, religion or  The Global Antibiotic Research and
culture, who are united by community Development Partnership (GARDP) is a
of interest, respect for the rule of law
not-for-profit organization developing new
and individual rights and freedoms, and
treatments for drug-resistant infections
by the pursuit of the positive ideals of
that pose the greatest threat to health.
parliamentary democracy.
 Established by the WHO and the Drugs for
 The Association is made up of over 180
Neglected Disease initiative (DNDi) in 2016,
Legislatures (or Branches) divided up
between nine geographic regions of the GARDP is a core element of WHO’s Global
Commonwealth. Action Plan on Antimicrobial Resistance.

It offers a vast opportunity for  The Global Antibiotic Research &


O


Development Partnership (GARDP)
C

Parliamentarians and parliamentary staff


S.

to collaborate on issues of mutual interest accelerates the development and access of


TE

and to share good practice. treatments for drug-resistant infections.


O
FN

32 PPP-PTS-4392/082023/17
D
.P
W
W
W
https://www.pdfnotes.co/

 Together with private, public and non-  The DSN also provides radar and radio
profit partners, GARDP works to preserve astronomy observations that improve our
the power of antibiotics for generations to understanding of the solar system and the
come. larger universe.
Why in News?  The DSN is operated by NASA’s Jet
 Bugworks is currently developing a Propulsion Laboratory (JPL), which also
compound that aims to address problems operates many of the agency’s interplanetary
arising from serious hospital and community robotic space missions.
infections.  The DSN consists of three facilities spaced
equidistant from each other – approximately
93. Correct Option: (b) 120 degrees apart in longitude – around the
world. These sites are at Goldstone, near
Explanation: Barstow, California; near Madrid, Spain;
 Option (b) is correct: Mylara Lingeshwara and near Canberra, Australia.
Temple is a Hindu temple dedicated to the  The strategic placement of these sites
god (Mailara dynasty), a form of the god permits constant communication with
Shiva in Mylara. Mylara is the main deity
spacecraft as our planet rotates – before a
of this temple, who is an incarnation of Lord
distant spacecraft sinks below the horizon
Shiva.
at one DSN site, another site can pick up

RE
Supplementary notes the signal and carry on communicating.
Mylara  The antennas of the Deep Space Network
are the indispensable link to explorers
 Mylara is a folk deity identified as a
manifestation of Lord Shiva. This deity, venturing beyond Earth. They provide the
commonly known as Mailara in Karnataka crucial connection for commanding our
and as Khandoba in Maharashtra, has a spacecraft and receiving their never before
plethora of other names as well, such as seen images and scientific information on
Khanderao and Khandnatha.
O Earth, propelling our understanding of the
universe, our solar system and ultimately,
 This cult is prevalent in the southern part of
India, including Karnataka, Maharashtra, our place within it.
Andhra Pradesh, and Tamil Nadu. Why in News?
SC
Why in News?  NASA and the European Space Agency
 The recent discovery of two sculptures, have been supporting ISRO to monitor the
which resembles one belonging to 15th Chandrayaan-3.
century A.D. and another to 17th century
A.D., at Basrur, near Kundapura, in Udupi
district has proved that the ancient Mylara 95. Correct Option: (c)
cult existed in the coastal region.
Explanation:
GS

 All statements are correct.


94. Correct Option: (b)
Supplementary notes
Explanation
Global Biodiversity Framework Fund
 Statement 3 is incorrect: The DSN
consists of three facilities spaced equidistant (GBF-Fund)
from each other – approximately 120  The Seventh Global Environment Facility
degrees apart in longitude – around the (GEF) Assembly in Vancouver, Canada,
world. These sites are at Goldstone, near established a Global Biodiversity Framework
Barstow, California; near Madrid, Spain; Fund (GBF-Fund).
and near Canberra, Australia.
 GBF-Fund is a special trust fund under
Supplementary notes the GEF, to support the implementation of
NASA Deep Space Network the Kunming-Montreal Global Biodiversity
Framework agreed at COP15 last year.
 NASA’s Deep Space Network is the
largest and most sensitive scientific Why in News?
telecommunications system in the world.
 The Global Biodiversity Framework Fund
The Deep Space Network - or DSN - is
(GBFF) was finally ratified and launched
NASA’s international array of giant radio
O

at the Seventh Assembly of the Global


C

antennas that supports interplanetary


S.

spacecraft missions, plus a few that orbit Environment Facility (GEF) in Vancouver,
TE

Earth. Canada.
O
FN

PPP-PTS-4392/082023/17 33
D
.P
W
W
W
https://www.pdfnotes.co/

96. Correct Option: (b) 98. Correct Option: (c)


Explanation: Explanation:
 Statement 2 is incorrect: While the Suez  Statement 3 is incorrect: The geographical
Canal route between Europe and Asia has a proximity of the GCC states and the
distance of 21,000 kilometres, the NSR (in similarity of their regulations and economic
red) has a distance of 13,000 kilometres. The and social conditions were additional
NSR represents a shortcut for the transfer factors that helped in the establishment of
of goods between Europe and Asia and thus the GCC.
offers significant cost savings for shipping
companies. Supplementary notes

Supplementary notes Gulf Cooperation Council


 GCC is a union of six countries in the Gulf
Northern Sea Route
region, Saudi Arabia, the UAE, Qatar,
 The Northern Sea Route (NSR), the shortest Kuwait, Oman and Bahrain. The council is
shipping route for freight transportation the largest trading bloc of India.
between Europe and countries of the Asia-
Pacific region, straddles four seas of the  The GCC is a political, economic, social,
Arctic Ocean. Running to 5,600 km, the and regional organisation according to
Route begins at the boundary between the its charter. It is a regional co-operation
system between the Arab States of the Gulf

RE
Barents and the Kara seas (Kara Strait)
and ends in the Bering Strait (Provideniya created to meet the challenges imposed by
Bay). surrounding circumstances.
 Murmansk, popularly called the capital of  All current member states are monarchies,
the Arctic region and the beginning point of including three constitutional monarchies
the Northern Sea Route (NSR), is witnessing (Qatar, Kuwait, and Bahrain), two absolute
the rising trend of Indian involvement in monarchies (Saudi Arabia and Oman), and
one federal monarchy (the United Arab
O
cargo traffic.
Emirates, which is composed of seven
Why in News? member states, each of which is an absolute
 India is keen to collaborate on a partnership monarchy with its own emir).
regarding the development of the NSR  The GCC was established on 4 February
SC
recognising the potential it holds for 1981 and held its first summit meeting on
enhanced connectivity and trade. 25 May 1981 in Abu Dhabi, United Arab
Emirates. Summits have been held every
97. Correct Option: (b) year since then.
Why in News?
Explanation:
 India is seeking greater trade engagement
 Statement 2 is incorrect: It is a metal art.
It originated in the kingdom of Bidar under with the Gulf Cooperation Council.
GS

the rule of Bahamani Kingdom.


Supplementary notes 99. Correct Option: (b)
Bidri craft Explanation:
 The Bidri Craft originated in Hyderabad of  Statement 2 is incorrect: EEE is only
Andhra Pradesh. It involves silver work on spread to humans through the bite of an
metals, which is intricate. infected mosquito. EEE is not spread person-
to-person, people to animals, or animals to
 Bidri is engraved with a sharp chisel on a
people.
metal base, which can be an alloy of zinc,
copper, tin and lead. Supplementary notes
 Silver wires are wrapped on the engraved Eastern equine encephalitis (EEE)
patterns by hammering. Some of the
bidriware items are Hukkah bases, trays,  Eastern equine encephalitis (EEE) is
spoon, vases, boxes of various sizes,paper an extremely rare but serious and often
cutters and pencil boxes, ashtrays and fatal infection that causes encephalitis or
buttons inflammation of the brain.

Why in News?  It is spread by the bite of a mosquito infected


with EEE virus (EEEV).
O

Prime Minister Narendra Modi gifted Bidri


C


EEEV can also infect a wide range of
S.

work pair of ‘Surahi’ from Telangana to 


animals including mammals, birds,
TE

South African President Cyril Ramaphosa.


O
FN

34 PPP-PTS-4392/082023/17
D
.P
W
W
W
https://www.pdfnotes.co/

reptiles, and amphibians. The spread of diurnal i.e active during the daytime. Some
EEEV to mammals (including humans and raptors are crepuscular, which is active
horses) occurs through the bite of infected primarily during twilight.
mosquitoes that feed on both birds and
 Raptors have a distinguished style of
mammals.
hunting. They use stealth and speed to
Why in News? sneak up on their prey, and once in range,
 Rare mosquito-borne illness eastern equine pursue with a great burst of speed, often
encephalitis reported in Alabama and New weaving deftly through tree branches and
York, causes one death. other obstacles while keeping their eyes
trained on their quarry.
There are some small to medium sized birds
100. Correct Option: (b) 
of prey like Falcons and Baza. These feed
Explanation: upon small mammals, insects, reptiles,
 Statement 3 is incorrect: Raptors are small birds and carrion.
mostly diurnal i.e active during the daytime.  Large birds of prey include Eagles,
Some raptors are crepuscular, which is Vultures, Hawks, Osprey. They mainly feed
active primarily during twilight. on insects, birds, reptiles, medium-sized
Supplementary notes mammals, amphibians, fish, crabs, with

RE
some on carrion.
Raptors
Why in News?
 Bird of prey is known as the Raptor. Raptors
are distinguished by keen vision, powerful  India’s raptors (birds of prey) are declining
talons and beaks. Keen vision helps them in fast and the reasons for the decline are
finding food during flight, talons or strong poorly understood, according to the State
feet are for holding food, and a strong curved of India’s Birds 2023: Range, trends, and
beak for tearing flesh. Raptors are mostly
O conservation status released recently.
SC
™™™™™
GS

O
C
S.
TE
O
FN

PPP-PTS-4392/082023/17 35
D
.P
W
W
W

You might also like